Constitutional Law In Class Quizzes

Pataasin ang iyong marka sa homework at exams ngayon gamit ang Quizwiz!

While vacationing, an African-American man entered a private restaurant in a remote town. All other customers in the restaurant at the time were white males who lived within a few miles of the restaurant. The man was refused service. The restaurant purchases a substantial portion of its food from local suppliers, who obtain their goods from a variety of sources, both in-state and out-of-state. The state has no statute prohibiting this type of discrimination. Congress, however, has passed a statute prohibiting restaurants from refusing service based on race and allowing private actions for damages for violations of the statute. Assuming the man can establish a statutory violation, what is the likely outcome of the suit? A) The man likely will prevail, even if Congress had not acted, because this constitutes a denial of equal protection of the laws under the Fourteenth Amendment. B) The man likely will prevail, even if Congress had not acted, because the activities of the restaurant violate the privileges and immunities granted to all citizens under the Fourteenth Amendment. C) The man likely will prevail, because Congress has enacted the statute. D) The man likely will not prevail, because this is local activity which cannot be regulated under the commerce power.

C

The Constitution provides that the President/Executive Branch: A) Can raise and spend money for the Country's General Welfare. B) Has plenary power over the US military. C) Has plenary power over foreign affairs. D) Is charged with executing the laws passed by Congress.

D

At a meeting with some of his advisors, the President lost his temper and loudly and unfairly criticized a position paper prepared by a female aide with whom he had had an affair just before he was elected President. He told the aide she was fired and would never work in government again "except maybe in the kitchen." The outrageous remarks caused the aide severe emotional distress. She brought suit for intentional infliction of emotional distress. If the President moved to dismiss, what would be the most important criterion that the court would use to decide the motion? A) Whether the President's actions were predicated on his official acts. B) Whether the President would be hampered in the conduct of his official duties in defending the lawsuit. C) Whether the separation of powers doctrine is served by postponing the case until the President left office. D) Whether the President's affair with a subordinate constituted a crime.

A

A state statute prohibits merchants from selling goods manufactured in a foreign country unless the merchant clearly marks the goods with their country of origin. The United States has a treaty with a foreign country that allows each country to import and sell goods and products from the other country. If a person is prosecuted under the state law for refusing to mark goods as being of that foreign country's origin, which of the following statements reflects the most likely outcome of the case? A) The person should be acquitted because the state statute is unconstitutional. B) The person will be acquitted, but only if the treaty with the foreign country preceded the state statute in point of time. C) The person will be found guilty because the treaty with the foreign country is no defense to a criminal prosecution in state courts for violating state laws. D) The person will be found guilty because the treaty is no defense to the criminal prosecution in the absence of effectuating legislation by Congress.

A

A tire company is the manufacturer of, and owns the patent for, a new and unique type of truck tire. The company is the largest employer in the state in which it is located. That state enacted a law requiring all trucks using its highways to be equipped with the company's unique truck tires. An out-of-state trucking company filed suit against the state, and eventually the United States Supreme Court held that the state's statute violated the Commerce Clause. Subsequently, Congress enacted a law that required all trucks engaged in interstate commerce to use the company's unique tires. Is this federal law constitutional? A) Yes, because Congress's control over interstate commerce is very broad. B) Yes, only if Congress can demonstrate that the use of tires of another design somehow unduly burdens interstate commerce. C) No, if most states permit tires of another design. D) No, because the federal law reverses a decision of the Supreme Court, and the Supreme Court is the sole arbiter of constitutional issues.

A

An ordinance of a city prohibits leafleting on the grounds of any hospital or on the sidewalks within five feet of the hospital entrances during visiting hours. A member of a religious group advocating prayer to restore the sick to good health is arrested for violating the ordinance, is fined $100, and is convicted. She appeals her conviction, claiming that her constitutional rights were violated. The case was heard by the state supreme court, which ruled that while the ordinance was permissible under the United States Constitution, it was unconstitutional under the state constitution because the fine money was designated to go to the city's only hospital, which was privately owned, rather than to the city. The city seeks to bring the case before the United States Supreme Court. Should the United States Supreme Court grant certiorari? A) No, because the case was decided on independent state grounds. B) No, because the case is moot. C) No, because this is a political question. D) Yes, the Supreme Court should hear the case on its merits, because it involves an important federal question.

A

Congress approved an act that contained an appropriation of $1 million for a professor at a state university to study the effects of volcanic eruptions on the temperature of sea water. The bill contained a second appropriation of $1 million for a professor at another state university to study the effects of oil drilling on the population of bears in a national forest. The President drew a line through the first appropriation, with the intent of canceling the provision, and then signed the bill. The professor studying volcanic eruptions brought suit alleging that the action of the President was unconstitutional and therefore the striking of his appropriation was invalid. Was the President's action constitutional? A) No, because the President has no line item veto power. B) No, because the President may not veto an appropriation passed by Congress. C) Yes, because the President could have vetoed the appropriation if it were contained in a separate bill. D) Yes, because the President has the power to refuse to spend funds that have been appropriated unless the spending is specifically mandated.

A

Congress enacted a law setting forth new requirements concerning the permitted variance in the size of congressional districts. Because many of the congressional districts in which members of Congress had traditionally run would be changed by these requirements, and because Congress desired a speedy determination of the constitutionality of the statute, the statute specifically grants standing to any sitting Congress member to sue the relevant state redistricting commission to determine the constitutionality of the act as it affects that Congress member's district. The law provides that original jurisdiction for such suits is mandated in the United States Supreme Court. Is the portion of the statute establishing exclusive original jurisdiction in the United States Supreme Court constitutional? A) No, because it is inconsistent with Article III of the Constitution. B) No, because by granting standing only to sitting Congress members, it denies equal protection of the laws to other individuals who may wish to run for Congress. C) Yes, because it is authorized pursuant to the power of Congress to enforce the Fifteenth Amendment. D) Yes, because Congress has the power to determine the jurisdiction of federal courts.

A

Congress enacted a statute making it a federal crime for any person not licensed to carry a hand-gun to possess one on the premises of any public or private primary or secondary school. An agent of the Bureau of Alcohol, Tobacco, and Firearms was giving a talk to a group of seniors at a high school. He noticed a handgun in the pocket of a student attending the talk and, after finding that the student did not have a license, arrested him for violating the statute. At trial, the student moved for acquittal on the ground that the act is unconstitutional. How should the court rule on the motion? A) Grant the motion, because the statute does not substantially affect interstate commerce. B) Grant the motion, because the power to regulate conduct in public schools is reserved to the states under the Tenth Amendment. C) Deny the motion, because there is a rational relationship between the regulation of firearms at schools and interstate commerce. D) Deny the motion, because the firearm probably traveled in interstate commerce.

A

Congress enacted a statute that gives individuals the right to view any file that the government has assembled concerning that individual. The President was considering the appointment of a candidate as an ambassador and requested that appropriate government agencies conduct a background investigation. The investigation revealed some derogatory information, and the President declined to appoint the candidate. After his attempts to view the file were refused by White House staff, the candidate brought an action to inspect his personnel file under the aforementioned act. Which of the following presents the strongest argument that the statute is unconstitutional? A) As applied in this case, the statute restricts the President's right to frank and unfettered confidential advice. B) The statute interferes with the right of privacy of the informants. C) Congress does not have the power to give the public access to private information. D) The doctrine of sovereign immunity prevents the suit of the candidate.

A

In January, 2023, Congress passed a law that conditioned a state's receipt of 5% of federal highway funds on the state first passing a law that allows transgender individuals to participate in competitive sports in schools. Did Congress have authority to pass this law? A) No, because the statute's condition (on the state's behavior) is not sufficiently related to (there is an insufficient nexus to) the purpose for which the federal funds are allocated. B) Yes, because Congress is empowered to attach conditions on the funds it grants to states under the Taxing and Spending Clause of Article I. C) Yes, because highways are instrumentalities of interstate commerce. D) Yes, because the Taxing and Spending Clause is an independent source of federal power from the Interstate Commerce Clause.

A

Anita Baker is a woman from Tulsa Oklahoma who became pregnant on March 20, 2023. She discovered her pregnancy by chance on March 30, 2023. Anita Baker had an abortion performed April 2, 2023, approximately 13 days after her egg was fertilized. Oklahoma has a law prohibiting abortion any time after fertilization and defines it as murder, a capital crime ("Oklahoma Abortion Law'). Ms. Baker was immediately arrested after someone informed the police that she had an illegal abortion. She and her doctor now face life imprisonment or the death penalty. What is the BEST argument you can make that the Oklahoma Abortion Law is unconstitutional? A) Under Dobbs v. Jackson, abortion may be prohibited, but the death may not be imposed for having an abortion. B) The penalty imposed by the law constitutes cruel and unusual punishment. C) Imposing the death penalty for having, or assisting, an abortion violates the Privileges or Immunities Clause of the US Constitution. D) Dobbs v. Jackson abolished abortion in the United States, it did not impose the death penalty.

B

The President of the United States accepted an invitation to give a commencement address at a small, Midwestern university in its auditorium. Pursuant to school rules, no one is permitted to bring posters, banners, or signs of any kind larger than the size of a piece of notebook paper into any event at the auditorium. The main purpose of the rule is to prevent obstruction of the view within the auditorium. Nevertheless, at the commencement ceremony, a student in a front row balcony seat unfurled a banner that he had hidden in his coat with a message supporting the President. The student was promptly arrested and charged in municipal court with trespassing. The student filed suit in federal court to enjoin the municipal prosecution and to have the trespass ordinance declared unconstitutional as applied to him. Will the federal court likely hear the student's case? A) Yes, if he argues that the trespass ordinance is invalid on its face. B) Yes, if he argues that the prosecutor had no hope of conviction and was proceeding to harass him. C) No, if the prosecution argues that the student lacks standing. D) No, if the prosecution argues that the case involves a political question.

B

Under a state's law, public schools in each school district are funded by a percentage of the property taxes collected in that district. The system was adopted because it allows communities to decide how much to invest in education, bringing government decision-making to a local level. Several school districts in the state are comprised almost entirely of low-income minority residents, and property tax revenue in those districts is much lower than in wealthier districts. As a result, funding for public schools is much lower in those districts. The parent of a child who attends school in low-income district brought an action against state officials seeking to enjoin the state school-funding law and force state-wide equitable funding of public schools, arguing the current funding system violates the Equal Protection Clause of the Fourteenth Amendment. Should the court uphold the state school funding law? A) Yes, because the law is substantially related to an important state interest. B) Yes, because the law is rationally related to a legitimate state interest. C) No, because the law is not necessary to achieve a compelling interest. D) No, because the system discriminates based on a suspect classification.

B

Which of the following statements is the BEST answer regarding Carolene Products? A) In the Carolene Products footnote the Supreme Court embraced a deferential approach to examining statutes that affected groups that had historically experienced discrimination. B) In the Caroline Products footnote the Court articulated situations where the Court should scrutinize statutes more rigorously than the very deferential level of scrutiny that the Court normally applied to statutes. C) The Carolene Products footnote was designed to eliminate the lowest level of scrutiny (rational basis) for all laws affecting all Americans. D) Classes that are deemed "suspect" are characterized by qualities that are undesirable.

B

Which of the following statements is the BEST answer regarding Schuette (Michigan's statewide ban on affirmative action) and Romer v. Evans? A) The Court struck down the statewide ban on affirmative action in Schuette for the same reasons the Court struck down the state ban on legislation affecting gay people. B) One way to distinguish Schuette from Romer v. Evans is that Schuette banned affirmative action, which the Court considered discrimination, and in Romer, the Court banned a state law targeting gay people out of animus. C) One way to distinguish Schuette from Romer is that Schuette permitted affirmative action at the local level. D) The Court struck down the state ban on affirmative action in Schuette because racial discrimination is subject to strict scrutiny whereas discrimination against gay people is not.

B

Which of the following statements is the BEST answer regarding the "Dormant Commerce Clause?" A) The Dormant Commerce Clause is found in the text of Article III of the US Constitution. B) The "Dormant Commerce Clause" does not exist in the actual text of the Constitution. C) The Dormant Commerce Clause is found in the text of Article II of the US Constitution. D) The Dormant Commerce Clause is found in the actual text of Article I.

B

Which of the following statements is the BEST answer regarding the Constitutional bases for Congressional Acts? A) Congress can act when such action is necessary and proper to achieve a goal that is in the General Welfare of the United States. B) Congress may enact legislation whenever such legislation does not violate the powers of the sovereign states. C) Several of the major foundations for Congressional action are: (1) the Enabling Clauses of the Civil War Amendments; (2) the Commerce Clause; and (3) the Spending Clause D) The TEXT of the Enabling Clauses of the Civil War Amendments are much more limited than the Necessary and Proper Clause of Article I.

C

Which of the following statements is the BEST answer, assuming the Court's current interpretation of the 10th Amendment? A) A law that requires state officials to report the number of hate motivated crimes in their state is most likely a violation of the 10th Amendment. B) In the case City and County of SF v. Trump the Court held that the President's denial of federal funds based on the refusal of states to cooperate with federal immigration authorities violated the Spending Clause. C) In the case City and County of SF v. Trump the Court held that the actions by the Trump administration violated, among other things, the 10th Amendment. D) In the case City and County of SF v. Trump the Court held that it was a violation of the 10th Amendment for San Francisco to refuse to cooperate with federal authorities in the enforcement of federal law.

C

Which of the following statements is the BEST answer? A) In Romer v. Evans, the COLORADO Supreme Court initially found the anti-gay referendum (Amendment 2) constitutional since it did not substantially impede the political process. B) The ""rational basis with bite"" standard is a heightened level of scrutiny because it requires that the means employed by the legislation must be more narrowly tailored to accomplish the state's goal than with traditional rational basis. C) The wording of the "Rational basis with bite"" test is the same as that of the traditional rational basis test (Rationally related to a legitimate government purpose) but simply refuses to recognize animus as a legitimate government purpose/goal. D) In Cleburne, involving a zoning ordinance that presumably discriminated against the mentally disabled, the Court utilized strict scrutiny because of the animus evidenced by the zoning ordinance.

C

Which of the following statements is the best answer with respect to the Privileges & Immunities Clause? A) Under the Privileges & Immunities Clause, Camden, NJ can require that its firefighter come from only Camden. B) Florida cannot charge a higher park entrance fee for out of state residents than for Florida residents. C) It would not violate the Privileges and Immunities Clause if Florida banned all professional gamblers who made their living from gambling from Florida casinos. D) Under the Privileges and Immunities Clause, Florida can prohibit out of state residents from practicing law in Florida, even if they've passed the Florida bar.

C

In January, 2010, Congress passed a statute titled the "Child Abuse Prevention Act" or "CAPA" in which Congress provided a federal private cause of action for the families of children whose children (under the age of 8) were violently sexually molested or exposed to other extremely harmful sexual behavior by an adult. CAPA permitted the plaintiffs to seek monetary damages against the abuser. Which is the BEST answer regarding whether Congress had constitutional authority for passing the CAPA? A) Congress does not have the authority to pass CAPA because child molestation is not a serious national problem. B) Congress cannot pass CAPA because, under the text of the Constitution, family violence is an area involving only the state police power. C) Congress can pass CAPA because child molestation is a serious issue which, in the aggregate, affects the General Welfare of the United States. D) There is no authority under the Constitution for Congress to pass this law.

D

Intending to encourage long-time resident aliens to become American citizens, a state passed a law denying numerous state and municipal jobs to persons who had been resident aliens for longer than 10 years. Those already in the state had to apply for American citizenship within a year after the law took effect. Persons who had acquired resident alien status prior to achieving the age of majority had until age 30 to acquire such status or be automatically disqualified from obtaining such a job. A 40-year-old man who has been a resident alien in the state for 15 years applied for a job as a police emergency response telecommunications expert. He had not filed for citizenship within the one-year grace period. May the state constitutionally rely on the statute to refuse to hire the man? A) Yes, because a police department performs an integral governmental function and the state law does not discriminatorily classify resident aliens by race or ethnicity. B) Yes, because aliens are not entitled to the privileges and immunities of state citizenship. C) No, because the law does not apply equally to all aliens. D) No, because the reasons for application of the law to the man do not appear compelling.

D

Which of the following statements is the BEST answer? A) Loving v. Virginia, involving miscegenation laws, was a case involving only Equal Protection and not Substantive Due Process. B) Justice Douglas, in his opinion for the Court in Griswold, based his opinion principally on a right to privacy based on the liberty interest in the Substantive Due Process Clause. C) The concurring opinions in Griswold v. Connecticut argued, among other things, that the 9th Amendment and Substantive Due Process provided a constitutional basis for a right to privacy with respect to contraception. D) One of the most important cases creating the modern Right to Privacy as a fundamental Right was Lochner v. New York.

C

A city passed an ordinance prohibiting all of its police officers and firefighters from "moonlighting" (working a second job). The ordinance was passed to ensure that all police officers and firefighters were readily available in case an emergency should arise and for overtime work when the situation warranted it. Other city employees, including members of the city council and the city manager, had no such restrictions placed on secondary employment. A police officer who wanted to moonlight as a dancer at a nightclub within city limits brought suit in federal court, alleging that the ordinance violated her constitutional rights. Will the court likely find the ordinance constitutional as applied to the police officer? A) No, because the ordinance restricts the officer's First Amendment rights to freedom of expression. B) No, because the singling out of police officers and firefighters violates equal protection. C) Yes, because the city has a significant interest that it seeks to regulate. D) Yes, because there is a rational basis for the ordinance.

D

A zoning ordinance in a town prohibits more than three unrelated individuals from living in a dwelling unit. A husband and wife who live in a five-bedroom home provide foster care for children who are temporarily without parents. At a time when there were six foster children living in their house, a zoning officer cited the husband and wife for violation of the ordinance. The couple brought suit in federal court challenging the constitutionality of the ordinance. When the case came to trial, there was only one foster child living in their house. Consequently, the town revoked the citation and moved to dismiss the complaint, arguing the matter had been resolved. The couple objected to the motion. How should the court rule on the town's motion? A) Grant the motion, because the plaintiffs no longer are in violation of the ordinance. B) Grant the motion, because constitutional issues are no longer ripe for review by the court. C) Deny the motion, because once a constitutional issue is raised, a change of circumstances cannot defeat the constitutional claim. D) Deny the motion, because the plaintiffs may in the future want to take care of more than one foster child at a time in their home.

D

A Congressman was indicted for conspiracy to use improper influence to prevent a tax fraud investigation of his ex-law partner. The prosecution desires to show that, as part of the conspiracy, the Congressman made a speech on the floor of the United States House of Representatives at a relevant time, urging a cutback in appropriations for the Internal Revenue Service. The Congressman's legislative aide and speechwriter has been called as a witness to the circumstances of the preparation of that speech. The Congressman objects to the calling of his speechwriter. How should the court rule on the objection? A) Deny the objection, because the Speech or Debate Clause does not protect speeches when they are the subject of a criminal prosecution. B) Deny the objection, because the Speech or Debate Clause only protects speeches made on the floor of Congress, not relations with congressional aides. C) Sustain the objection, because the Speech or Debate Clause protects all confidential communications between a Congressman and his aide. D) Sustain the objection, because the aide is protected by the Speech or Debate Clause when preparing a speech for the floor of Congress.

D

A city condemned all of the buildings in a decaying warehouse district and offered them to developers at no cost, provided the developer submits a building revitalization plan to the city's planning commission, the commission approves the plan, and the developer pays the city's redevelopment authority to perform the work needed to revitalize the building. An experienced developer submitted a plan to revitalize a block-long building, and his plan was approved by the planning commission. However, the developer wanted to perform the revitalization work himself, both to ensure that the job was done correctly and to save a substantial amount of money. Although he offered to post a surety bond to insure his performance, the city refused his offer. As applied to the contractor, is the city's requirement that the developer use the city's redevelopment authority constitutional? A) No, because it violates the privileges and immunities protected by Article IV. B) No, because it constitutes an improper exaction under the Fifth Amendment. C) Yes, because the city is seeking just compensation for the building that it would provide to the developer. D) Yes, because the requirement is rationally related to a legitimate government interest.

D

A city has many parks, as well as a country club with a golf course. While the city's parks are accessible to all without a fee, the city charges a $1,000 application fee and $100 per month dues to belong to the country club. A resident of the city wishes to join the country club but cannot afford the application fee or monthly dues. If the resident brings suit against the city on the ground that the fee and dues discriminate against the poor in violation of the Equal Protection Clause, who likely will prevail? A) The resident, because a person cannot be deprived of a public right or benefit on the basis of inability to pay. B) The resident, because the poor qualify as a protected class. C) The city, because only de jure discrimination against the poor has been held to violate the Equal Protection Clause. D) The city, because the membership privilege is not an important enough deprivation.

D

A cattle-producing state adopted a statute requiring any food service business operating in the state to serve beef raised in the United States. A licensed hot dog vendor who worked at a football field within the state and who had been buying hot dogs made with foreign beef for the past several years estimated that switching to an all-beef hot dog made from United States beef would reduce his profits by 10%. An attorney hired by the vendor to challenge the statute discovered during research into the case that most of the footballs used at the football field at which the vendor worked were made of foreign leather. Which of the following grounds is the vendor's best argument against the constitutionality of the state statute? A) The statute burdens foreign commerce. B) The statute violates equal protection guarantees because it is not rational to prohibit the sale of foreign beef but not foreign leather. C) The statute substantially interferes with the vendor's right to earn a living under the Privileges or Immunities Clause of the Fourteenth Amendment. D) The statute constitutes a taking without due process of law.

A

A city entered into a contract with a cable and Internet provider allowing the provider to be the exclusive cable and Internet provider in the city in exchange for certain rights. To facilitate installation of the provider's fiber optic cables for all of the city's residents, the city passed an ordinance requiring all apartment owners to allow the provider to install cables in their buildings. Installation of the cables involved drilling a hole in one exterior wall of the building and running the cables between the interior walls of the buildings. The owner of an apartment building within the city did not want to allow new cables to be installed in his building, as he had gone through a similar process with another company three years earlier and he believed that the other company had damaged his property. To deter the installation, the owner filed suit against the city, claiming that the ordinance amounts to a taking under federal law. How should the court rule? A) For the owner, because the ordinance amounts to a taking without just compensation. B) For the owner, because the government does not have a right to require owners of private property to allow private installation of cable lines. C) For the city, because the ordinance is a regulation and not a taking. D) For the city, because any taking here is de minimis.

A

A state suffering from a severe loss of tax revenues, due to an initiative that cut state sales taxes in half, enacted legislation that ended cost-of-living increases in all state employees' pensions. A state organization of employees brought suit against the appropriate state official in the federal court to reinstate the increase. What is the most likely result? A) The employees' organization will prevail, because the statute violates the prohibition against the impairment of the obligations of contracts by a state. B) The employees' organization will prevail, if it can show that the statute violates the state's constitution. C) The employees' suit will be dismissed, because the Eleventh Amendment prohibits a state's citizens from suing a state official for official acts in a federal court. D) The employees' organization will not prevail, because the state always has the power to amend its own legislation.

A

An attorney was employed by the United States Department of Health and Human Services in a regional office located in a tobacco-growing state. A labor contract between the agency and the clerical workers union contained a policy providing for termination of union employees only for certain specified grounds. The attorney, however, was not a member of the union and not covered by such a policy. The attorney was angered by the regional director's refusal to adopt a no-smoking policy for employees and visitors in the office. She posted a notice in the employee cafeteria ridiculing what she called the hypocrisy of an agency promoting health issues and nonsmoking programs while refusing to provide its employees with those same opportunities. The notice prompted a great deal of debate among the employees and was brought to the attention of the regional director, who was very displeased. Which of the following statements is most accurate regarding the director's right to dismiss the attorney? A) The attorney has a liberty interest in the exercise of her First Amendment rights that entitles her to a hearing to contest the grounds of her dismissal. B) The attorney has a property interest as a public employee that precludes her from being fired without notice and an opportunity to respond. C) The attorney has no right to a hearing because her statements were not an expression of views on public issues. D) The attorney has both a liberty interest and a property interest that entitles her to a pre-termination evidentiary hearing.

A

An employee of the United States Department of Labor was instructed by his superior to solicit subscriptions to the Department's bulletin on a door-to-door basis in the city in which he worked. While doing so, the employee was arrested for violation of a city ordinance that prohibited commercial solicitation of private residences. What is the employee's best defense? A) Intergovernmental immunity. B) The First Amendment freedom of expression as it applies to the states through the Fourteenth Amendment. C) The Equal Protection Clause as it applies to the states through the Fourteenth Amendment. D) The city ordinance effectively restricts interstate commerce.

A

Which of the following statements is the BEST answer with respect to Exceptions to the Dormant Commerce Clause? A) An example of a marketplace exception is if a state requires that all Floridians buy pencils only from Florida. B) If Congress approves of discrimination by states against other states, the Dormant Commerce Clause does not apply. C) If Congress approves of states discriminating against each other in a particular situation, there is no other Constitutional provision that could render that discrimination unconstitutional. D) The marketplace exception would not apply to a Broward County School that only bought local produce for its school lunches, and therefore its policy would violate the Dormant Commerce Clause.

B

Which of the following statements is the BEST answer? A) Congress may not, under its Spending Power, induce states to regulate matters that Congress may not otherwise regulate. B) Congress may not directly regulate the behavior of individuals under the Spending Clause. C) Congress may use its Spending Power to coerce states or individuals to do something or not do something, as long as it does so only through the spending of money. D) Congress may use its Spending Power to coerce states to pass laws, as long as Congress does not regulate individuals.

B

Which of the following statements is the correct answer regarding the 10th Amendment? A) Congress cannot pass a law prohibiting states from selling information regarding its drivers to third parties. B) The 10th Amendment does not grant states any specific exclusive powers. C) The 10th Amendment gives the states exclusive authority to legislate with respect to certain issues expressly delegated to them under the Amendment. D) The only circumstances when Congress may NOT legislate is with respect to issues granted to the states under the 10th Amendment.

B

Which of the following statements is the BEST answer regarding the Supreme Court's decision in Nat'l Fed'n of Index. Businesses v. Sebelius, dealing with the Affordable Care Act (ACA)? A) The Court ruled that Congress had no power to expand medicare under the Spending Clause. B) The Court ruled that the ACA's withholding of ALL federal medicare funds (including medicare funding authorized in previous years) if a state did not expand medicare under the ACA was unconstitutional. Congress certainly has the power to expand federal medicare aid to states under the Spending Clause, but it can't withhold previously authorized medicare funds if a state did not expand medicare coverage pursuant to the ACA. C) The Court ruled that the Affordable Care Act was unconstitutional as a violation of both the Commerce Clause and the Spending Clause. D) The Court ruled that imposing an individual mandate on individuals to purchase healthcare violated the Spending Clause.

B

A housing development contained one-, two-, and three-bedroom units. All units were suitable for occupancy, and the developers of the project filed the appropriate documents, including a Declaration of Restrictions that limited ownership and occupancy of the units to families or to groups of unrelated adults of not more than three in number. Each deed to the individual units also contained the restriction. One of the two-bedroom units was purchased by a woman and her boyfriend. They immediately moved into the unit with another unmarried couple who were friends of theirs. Other unit owners brought suit against the woman and her boyfriend to enjoin the occupancy by the other couple. If the other unit owners prevail, what will likely be the reason? A) The litigants are private parties and the restriction was not enacted by the government. B) Enforcement of the restriction is rationally related to a legitimate government interest. C) Notice was not given by the woman and her boyfriend to the sellers of the unit that they intended to occupy the residence with another couple. D) The restriction constitutes a lawful restraint on alienation.

B

Which of the following statements is the BEST answer regarding the Right to Travel? A) The most important doctrine preventing obstacles to the movement of people has been the Dormant Commerce Clause. B) The Right to Travel is considered a fundamental right by the Supreme Court under various constitutional doctrines, and most recently in Saenz v. Roe using the Privileges OR Immunities Clause C) The Right to Travel is considered a fundamental right by the Supreme Court under various constitutional doctrines, and most recently in Saenz v. Roe using the Privileges AND Immunities Clause. D) Miscegenation laws were ultimately struck down under the Privileges and Immunities Clause because they prevented mixed race couples from traveling throughout the United States.

B

A state adopted legislation making it a crime to be the biological parent of more than two children. The stated purpose of the statute is to preserve the state's natural resources and improve the quality of life for the state's residents. A married couple has just had their third child. They have been arrested and convicted under the statute. Which of the following is the strongest argument for voiding the convictions of the couple? A) The statute is an invalid exercise of the state's police power because there is no rational basis for concluding that the challenged statute would further the government's stated interests. B) The statute places an unconstitutional burden on the fundamental privacy interests of married persons. C) The statute places too much discretion in state officials to determine who will be permitted to bear children. D) The statute denies married persons equal protection of law.

B

A state bans the use of disposable diapers to reduce the volume of nonbiodegradable material in its landfills. The ban was a boon for diaper services within the state, but many parents of young children were displeased with the use of conventional diapers. With support from retail establishments that lost business from the disposable diaper ban, a grass roots coalition formed to fight the ban funded a study showing that the trucks and cleaning supplies used by diaper services within the state harmed the environment more than disposable diapers. The coalition and retailers then filed suit seeking to have the ban on disposable diapers declared unconstitutional. If the court strikes down the statute, on which of the following constitutional provisions would its decision most likely be based? A) The Equal Protection Clause of the Fourteenth Amendment. B) The Due Process Clause. C) The Impairment of Contracts Clause. D) The Privileges or Immunities Clause of the Fourteenth Amendment.

B

A state enacted a statute prohibiting the use in any medical facility in the state of any blood or blood derivative that has not been tested and declared free of HIV antibodies. The statute requires the tests to be done at a testing facility located in the state and approved by the state health commissioner. An established, reputable hospital system has a large hospital in the state, as well as hospitals in several other states. The hospital system has a long-term contractual relationship with a large, state-of-the-art blood testing facility located in another state to perform blood tests for all of its hospitals. The hospital system petitioned the state health commissioner for approval to use blood tested at the out-of-state facility. The commissioner, citing the statute, refused. The hospital system has sued to enjoin the commissioner from preventing the use of blood tested for the HIV antibodies at the out-of-state facility, alleging that the statute, as applied to it, is unconstitutional. Which party is most likely to prevail? A) The plaintiff, because the statute violates the Contract Clause. B) The plaintiff, because the statute violates the Commerce Clause. C) The defendant, because the statute does not discriminate against out-of-state competition. D) The defendant, because the statute is valid under the police power since it is directly related to public health.

B

A state engaged a private company to run its lottery, but the state maintained close regulation of the manufacture and distribution of lottery equipment by the private company in order to prevent frauds. One lottery regulation required the company to submit to the state all applications of persons being considered for employment. The state ran background checks on the prospective employees to ensure that they did not have a criminal record. A prospective employee that did not pass the state background check could not be hired by the company. An employee of the company who had a poor work record and called in sick often was spotted by her supervisor dancing at a bar one evening after she had called in sick during the day. The supervisor immediately told the employee that she should consider herself terminated, although the employee tried to explain that she in fact had been sick that morning but began feeling well by mid-afternoon. A state law provided that employees of the state could not be fired from their positions except for cause. The woman sued in federal court, claiming that she was constitutionally entitled to a hearing to determine whether her supervisor had cause to fire her. If the court rules correctly, will it find the employee's termination to be constitutional? A) Yes, because no hearing was required since the supervisor witnessed the employee's misconduct. B) Yes, because the employer is free to fire employees at will. C) No, because it violates the employee's right to procedural due process. D) No, because of the state's regulation of the hiring process.

B

A state governor proposes to place a nativity scene, the components of which would be permanently donated to the state by private citizens, in the state capitol building rotunda where the state legislature meets annually. The governor further proposes to display the state-owned nativity scene annually from December 1 to December 31, next to permanent displays that depict the various products manufactured in the state. The governor's proposal is supported by all members of both houses of the state legislature. A citizen of the state brings an action in court, arguing that the display would violate the Establishment Clause. Is the suit likely to be successful? A) Yes, because the components of the nativity scene would be owned by the state rather than by private persons. B) Yes, because the nativity scene would not be displayed in a secular context. C) No, because the components of the nativity scene would be donated to the state by private citizens rather than purchased with state funds. D) No, because the nativity scene would be displayed alongside an exhibit of various products manufactured in the state.

B

A state statute has detailed classifications of civil servants for both state and city positions. It provides that all civil servants who have been employed for over 18 months may be dismissed only for "misconduct" and also requires that state and city agencies comply with all procedures set forth in any personnel handbook issued by that agency. The personnel handbook of the state tollway authority sets forth detailed procedures for dismissal of civil servant employees. The handbook provides that written notice of the grounds for dismissal must be given to the employee prior to dismissal, and that the employee must, on request, be granted a post-dismissal hearing within three months after the dismissal takes effect. An employee is entitled to present witnesses and evidence at the post-dismissal hearing, and is entitled to reinstatement and back pay if the hearing board decides that the employer has not shown by a preponderance of the evidence that the dismissal was justified. A state tollway employee who had been employed for three years recently was fired. After an investigation by state auditors, the employee was notified by registered letter that he was being dismissed because of evidence that he took bribes from construction firms in exchange for steering contracts to them. He was informed of his right to a hearing and requested one as soon as possible. Three weeks after his dismissal, the state personnel board conducted a hearing at which the employee denied the charges and presented witnesses to attest to his honesty. At the conclusion of the hearing, the board upheld his dismissal, finding that it was supported by a preponderance of the evidence. If the employee files suit in federal court challenging his dismissal on constitutional grounds, will he be likely to prevail? A) Yes, because the employee had a right to a pretermination hearing at which he could present witnesses to support his side of the story. B) Yes, because the employee had a right to have an opportunity to respond to the charges prior to his dismissal. C) No, because the state may establish the required procedures for terminating an interest that it created by statute. D) No, because the procedures taken for termination of the employee's job satisfied due process requirements.

B

A state supplies U.S. history textbooks to all public high schools located in the state. The state education department has promulgated a rule requiring all high school students studying U.S. history to cover their books with a state-supplied book cover that has the state motto "Live Free or Die" on the front and a picture of the state governor on the back. A student refused to place this cover on his history book and was disciplined for violating the education department rule. The student challenged the validity of his discipline in a proper court, claiming the requirement violated his constitutional rights. Is the student likely to prevail? A) Yes, because a state cannot prescribe the type of book cover to be placed on schoolbooks. B) Yes, because a state cannot force an individual to display a message. C) No, because the words on the book cover are merely the state motto. D) No, because the books are furnished at the state's expense.

B

A state with a number of automobile manufacturing facilities within its borders and a high unemployment rate because of declining sales of automobiles, especially ones built domestically, enacted a statute calling for a $100-per-car tax on all foreign-built automobiles sold within the state. The tax revenues were to be placed into a state fund to be used to retrain the state's unemployed automobile workers. A major automobile importer and dealership owner brings suit in federal district court seeking to halt the enforcement of the statute on constitutional grounds. Should the court find the statute to be constitutional? A) Yes, because it is a proper exercise of a state's rights under the Import-Export Clause. B) Yes, if consented to by Congress. C) No, because the statute violates the Privileges and Immunities Clause of Article IV. D) No, unless the state can show that the statute is necessary to promote a compelling state interest.

B

A town adopted an ordinance providing that a person must have been a resident of the town for at least one year to be eligible to vote in school board elections. A resident who moved to the town seven months ago attempted to register to vote in the school board elections scheduled for the next month. However, the town clerk refused to register the resident because he will not have resided in the town for a full year prior to the election. The resident filed a class action suit on behalf of all of the new residents of the town, challenging the validity of the one-year residency requirement. Which of the following statements is correct? A) If the resident's suit is not heard before the election, it will be dismissed as moot, because the resident will have met the residency requirement by the time of the next annual election. B) The resident will prevail even if the matter is not decided until after next month's election. C) As long as there is some legitimate purpose for the one-year residency requirement, such as the need to prepare voting lists, the residency requirement will be upheld. D) The resident will lose because one-year residency requirements have been held permissible restrictions on the right to vote in local elections.

B

As part of legislation enacted for the stated purpose of improving science skills of schoolchildren, Congress appropriated funds to permit public school teachers who had been certified by state school districts as science lab instructors to provide supplemental science instruction to any students in either public or private schools who did not have access to science lab resources. To help ensure content neutrality, the statute required the instructors coming to the private schools to use portable science labs supplied by the public school districts, which contained the equipment and experiments that the instructors used for the same purpose in the public schools. A citizens' group filed suit in federal district court to challenge the constitutionality of funding the science teachers for private schools, alleging that most of the private schools covered by the statute were religiously affiliated schools. No members of the group have any children in either public schools or private schools affected by the statute. How is the court likely to rule? A) Dismiss the case on the pleadings, because the citizens' group does not have a sufficient stake in the controversy to have standing to challenge Congress's expenditure, which was authorized under its power to spend for the general welfare. B) Decide the case on the merits in favor of the government, because the legislation defines the context in which instruction can be provided in private schools so as to avoid excessive government entanglement with religion. C) Decide the case on the merits in favor of the citizens' group, because the appropriation's primary effect advances religion in violation of the Establishment Clause of the First Amendment. D) Decide the case on the merits in favor of the citizens' group, because the court will presume that any instruction provided on the premises of a religiously affiliated school will be influenced by religion.

B

Congress has enacted a statute that requires all companies engaging in business with the federal government to enact certain affirmative action programs in hiring. A small diner has been providing catering services for a local branch office of the United States Department of Agriculture. At lunch one day, a senior enforcement officer of the Department of Agriculture happens to notice the racial makeup of the diner's workforce, and informs the diner that it is in violation of the affirmative action statute, and files charges against the diner. If the diner challenges the validity of the federal statute, what is the government's best response to the argument that Congress has exceeded its legitimate powers? A) The act is a valid enforcement of the Due Process Clause of the Fifth Amendment and a valid exercise of the Enabling Clause of the Thirteenth Amendment. B) The act is a valid exercise of the commerce power and a valid exercise of the Enabling Clause of the Thirteenth Amendment. C) The act is a valid enforcement of the Due Process Clause of the Fifth Amendment and a valid exercise of the commerce power. D) The act is a valid exercise of the federal police power.

B

The state passed a law stating that "only persons living with their parents or guardians who are bona fide residents of the state shall be entitled to free public education; all others who wish to attend public schools within the state may do so, but they must pay tuition of $3,000 per semester." A 15-year-old girl moved in with her friend so that she could attend the public schools in the state, and the state legislature passed the tuition statute just as she completed her junior year. The girl wants to complete her senior year in the state high school, but cannot afford to pay tuition. If the girl sues in federal court to strike down the tuition statute, is the court likely to rule that the statute is constitutional? A) Yes, provided that the state can show that the statute is necessary to promote a compelling state interest. B) Yes, unless the girl can show that the statute is not rationally related to a proper state interest. C) No, because it infringes on the girl's fundamental right to an education. D) No, because it interferes with the girl's fundamental right to interstate travel.

B

To protect an endangered species of salamander, Congress passed a civil statute prohibiting hiking and fishing in wetland areas located in a particular national park. Violators could be enjoined from use of the national park for one year. The law was to take effect on January 1 of the new year. After the law went into effect, a local resident was stopped by a National Park Service ranger and given a $100 citation for fishing in the wetland areas. As authorized by the statute, a federal district court also issued an injunction prohibiting the resident from fishing in the wetlands area for one year. The resident appealed the injunction, asserting that he was unaware of the penalty, but he was unsuccessful. A final judgment was rendered against him. During the next few months, several hundred violations of the statute occurred, and the population of endangered salamanders continued to decline. To better achieve its conservation goal, Congress voted to amend the statute. The new law expanded the time an injunction could prohibit use of national parks to two years, made such injunctions applicable to all national parks, and imposed a $1,000 fine for all violations. The new law was made effective retroactively to the same January 1 date. As a result of the new law, the resident was notified that the injunction against him would be extended to a total of two years and that he owed an additional $900 fine. The resident's attorney brought an action claiming that the new law was unconstitutional as applied. Will the resident's claim be successful? A) Yes, because the new law proscribed greater punishment for the violation than was prescribed at the time the violation was committed. B) Yes, because a final judgment was rendered against him on appeal of the district court's injunction. C) No, because Congress has the power to make substantive changes to federal civil statutes and direct the federal courts to apply those changes. D) No, because the Ex Post Facto Clause under Article I, Section 10 applies to the states, not to the federal government.

B

To provide low-cost housing to the unemployed, a city has a policy of leasing empty city-owned buildings to social agencies that promise to convert or rehabilitate the buildings into habitable, low-cost apartments and to pay the city 10% of any net profit made from rentals. A church entered into such an agreement with the city and converted one of the city's abandoned office buildings into 50 small, low-cost apartments. The lease agreement used by the church provides, among other things, that the lessee must affirm a belief in God. The lease agreement was submitted to the city for approval prior to its use by the church, and it was approved. On the first day that the church made the apartments available for rent, the plaintiff, an avowed atheist, applied to lease a unit. The plaintiff's application was denied for the sole reason that the plaintiff refused to affirm a belief in God. If the plaintiff brings suit against the church on the ground that the required affirmation of a belief in God violates the plaintiff's constitutional rights, who likely will prevail? A) The plaintiff, because denial of a lease to atheists has been held to hinder the free exercise of religion. B) The plaintiff, because the purpose and effect of the church's policy results in a violation of the Establishment Clause. C) The church, because freedom of religion is not protected against acts of private individuals or groups or a private institution. D) The church, because as an atheist, the plaintiff has no standing to challenge the lease requirement on religious grounds.

B

A federal statute prohibits discrimination based on age for all persons between the ages of 50 and 70 in matters relating to employment. A state constitutional provision requires that all state judges retire at the age of 65. A judge in the state trial court reached his 65th birthday, and the governor of the state named his replacement. The judge refused to retire and brought suit in federal court to enjoin the governor from replacing him on the bench. Who is likely to prevail in the lawsuit? A) The state, because the Tenth Amendment prohibits the federal government from interfering with matters central to the administration of state government. B) The state, because the Eleventh Amendment prohibits the suit. C) The judge, because the Supremacy Clause invalidates the state constitutional provision. D) The judge, because the Fourteenth Amendment Equal Protection Clause protects the judge from discrimination.

C

A state Occupational Health and Safety Board recently issued regulations valid under its statutory mandate requiring that all employers in the state provide ionizing air purification systems for all employee work areas. These regulations replaced previous guidelines for employee air quality that were generally not mandatory and did not specify the method of air purification used. The requirements regarding air purification systems are likely to be unconstitutional as applied to which of the following employers? A) A wholly owned subsidiary of a Japanese corporation with seven retail outlets within the state. B) The state supreme court, which recently completed construction of its new courthouse with a non-ionizing air purification system which the builder is contractually bound to maintain for the next three years. C) A United States Armed Forces Recruiting Center located adjacent to the state capitol building. D) A privately operated community service center funded by donations and constructed through use of a loan provided by the United States Veterans Administration and repayable to that agency.

C

A state enacted legislation providing for the testing of all high school students in the performance of certain adult-world tasks. In public schools, the test was administered at the school by government employees. In private schools—both religious and secular—the test was administered by school personnel, and the schools were reimbursed for the costs of administering the tests and reporting the results to the state. An association with appropriate standing filed suit seeking to enjoin the reimbursement to the religious schools, claiming that the reimbursement violates the Establishment Clause of the First Amendment to the United States Constitution. Should the trial court grant the injunction? A) Yes, because direct state payment to religious schools violates the Establishment Clause. B) Yes, because the state may not require the religious schools to conduct tests that violate their religious principles. C) No, because the reimbursement has a secular purpose that neither advances nor inhibits religion and that will not entangle the state in religion. D) No, because the tests do not involve matters of religion.

C

A state legislature passed a law requiring all employers operating in the state's oil and natural gas fields to give preference in hiring to residents of that state. The bill banned the hiring of nonresidents unless no other qualified person could be found to fill an oilfield or natural gas field position. Under prevailing economic conditions, which included a substantial decline in petroleum prices, the statute was tantamount to a total ban on hiring of nonresidents because there were so many unemployed oil and gas workers and little new exploration was taking place. The plaintiff was an experienced oilfield worker who was denied a job in the state because his permanent residence was in another state, even though he had worked in many states and foreign countries and his qualifications were better than anyone else applying for the job. The sole reason given for not hiring the plaintiff was the preferential hiring statute favoring state residents. The plaintiff filed suit in federal district court challenging the statute. Who should prevail? A) The state, because employment discrimination is only unconstitutional if it involves race, religion, alienage, or sex. B) The state, because the state's interest in hiring local residents outweighs the interest of nonresidents. C) The plaintiff, because the law denies him the privileges and immunities of state citizenship. D) The plaintiff, because the preferential hiring law impairs the plaintiff's rights under the Contract Clause of the federal Constitution.

C

A state statute makes it unlawful to willfully cause or permit the life or health of a child under the age of 18 to be endangered. The statute also makes it unlawful to willfully cause or permit a child to be placed in circumstances that endanger the child's life or health. A local prosecutor brought charges against parents who brought their child to their church's religious service in which venomous snakes were passed around. The parents were convicted and have brought an appeal on constitutional grounds. Which of the following best describes the likely result of the parents' appeal? A) Their convictions will be upheld because the parents lack standing to challenge the statute on "free exercise of religion" grounds. B) Their convictions will be upheld because the state's interest in regulating activities involving children necessarily outweighs any rights of members of a church under the Free Exercise Clause of the First Amendment. C) Their convictions will be reversed if it can be shown that the statute is being applied only to interfere with religion. D) Their convictions will be reversed because the freedom to engage in conduct connected with one's religion is absolutely protected under the First and Fourteenth Amendments.

C

A state statute permits the state to seize and dispose of real property that was used to commit or facilitate the commission of a felony drug offense. After a drug dealer's arrest for selling cocaine out of his home, a felony, the state instituted an action of forfeiture against the drug dealer's house and property. After notice to the drug dealer and a hearing, a judge granted the order and the state seized the property. Six months later, after the time for any appeals had expired, the property was sold at a public auction to a third party. It was only when the third party brought an action to quiet title that a bank holding a properly recorded mortgage on the drug dealer's property learned of the forfeiture. Because the bank's mortgage payments were automatically deducted from an account the drug dealer had under a different name, no one at the bank was aware that the property had been seized. The only notice provided to parties other than the drug dealer was a public notice published for three weeks in a general circulation newspaper. The bank defends the quiet title action on the ground that it did not receive the notice required under the United States Constitution to protect its interest in the property. If the court rules that the bank's rights under the Due Process Clause of the Fourteenth Amendment were violated by the state's seizure of the property, what is the most likely reason? A) In any judicial proceeding affecting rights to real property, a claimant is required to provide notice and an evidentiary hearing to all parties with a legal interest in the property before taking actions affecting their rights. B) The government itself was the party that seized the property, rather than a private party using governmental processes. C) The notice was not adequate under the circumstances to apprise a party with a properly recorded legal interest in the property. D) The jurisdiction treats the mortgagee as having title to the property rather than merely a lien.

C

After a state legislature is presented with compelling evidence that insurance companies that offer burglary insurance policies are charging people different premiums based on race, the legislature passes a statute requiring every insurance company offering burglary insurance policies in the state to charge a uniform rate for customers who reside in the same county in the state. As long as it complies with this requirement, a company is free to charge whatever rate the market will bear for its burglary insurance policies. There are no federal laws covering uniform rates. An insurance company that offers burglary policies in the state but that is located in a neighboring state has filed suit in federal district court against state officials to challenge the statute on constitutional grounds. The insurance company wishes to charge customers residing within the same county varying rates for burglary insurance policies. What is the likely result of the suit? A) The statute will be held unconstitutional, because it is not necessary to achieve a compelling interest. B) The statute will be held unconstitutional, because it imposes an undue burden on interstate commerce. C) The statute will be held constitutional, because it is a reasonable exercise of the state's police power. D) The statute will be held constitutional, because it is narrowly tailored to achieve the compelling interest of remedying racial discrimination.

C

An expert in art history had worked at a county art museum for many years as its executive curator. The museum had a policy that once an employee worked at the museum for five years, the employee would not be dismissed from the job unless "for cause." The county decided to sell the museum to a private corporation which promised to double the size of the museum and greatly increase its funding. Two months after the museum's ownership changed hands, the curator was informed that he was being dismissed and would be replaced by an individual chosen by the corporation. The curator brought an action against the corporation in state court, claiming that termination of his position without the opportunity for a hearing violated his rights under the Due Process Clause of the Fourteenth Amendment and the Contract Clause. For which party should the court rule? A) The curator, because the Due Process Clause entitles him to an opportunity for a hearing before dismissing him. B) The curator, because dismissing him violated the Contract Clause. C) The corporation, because the curator's dismissal does not implicate either the Fourteenth Amendment or the Contract Clause. D) The corporation, because the right to pursue a livelihood is not a protected liberty interest.

C

Congress authorized $20 million to be granted to each of five southern states that had reported outbreaks of a harmful mosquito-borne virus. The states were required to use the money for both ground and aerial spraying in an attempt to eradicate the mosquito larva. The law provided that all federal environmental protection funding to the states would be contingent on the states implementing the mosquito spraying program. One of the states, where federal environmental protection funding constituted about 1% of the state's budget, determined that spraying swampland with the chemicals could have a wide range of unintended consequences, and the state refused to implement the program. As a result, the federal government terminated all of the state's environmental protection funding. The state then filed an action to force the federal government to continue to provide funding for the state's other environmental protection projects unrelated to the mosquito spraying program. Who will most likely prevail? A) The state, because Congress may not use its spending power to coerce states into implementing federal programs. B) The state, because controlling mosquito populations is not a regulation of interstate commerce. C) The federal government, because depriving a state of federal funding amounting to 1% of the state's budget is not unduly coercive. D) The federal government, because controlling mosquito populations in multiple states is a valid regulation of interstate commerce.

C

Congress passed legislation banning the hunting of snipe birds within the United States. The range of the snipe is quite limited; they are found primarily in only one state, although they migrate annually to several nearby states. Hunters from throughout the United States have traditionally traveled to the snipe's home state during snipe hunting season, bringing considerable revenue into the state. A state statute allows hunting of snipe during a two-week period in November and charges a $50 license fee for state residents and a $250 fee for hunters from other states. The bag limit is one snipe bird per licensed hunter. Is the state statute allowing snipe hunting valid? A) Yes, because states have the right to control their own natural resources and wildlife. B) Yes, because the power exercised is reserved to the states by the Tenth Amendment. C) No, because of the Supremacy Clause. D) No, because of the Commerce Clause.

C

Congress provided by statute that any state that does not enact a ban on texting while driving on state highways within the next three years shall be denied 10% of federal highway construction funding. What is the best argument that can be made in support of the constitutionality of this federal statute? A) The states ceded their authority over highways to the national government when the states accepted federal grants to help finance the highways. B) The federal government can regulate the use of state highways without limitation because the federal government paid for most of their construction costs. C) The requirement is not unduly coercive and is related to making travel on highways safer. D) A recent public opinion survey demonstrates that 90% of the people in this country support the ban.

C

Radon is a harmful gas found in the soil of certain regions of the United States. A state statute requires occupants of residences with basements susceptible to the intrusion of radon to have their residences tested for the presence of radon. The statute also provides that the testing for radon may be done only by testers licensed by a state agency. According to the statute, a firm may be licensed to test for radon only if it meets specified rigorous standards relating to the accuracy of its testing. These standards may easily be achieved with current technology, but the technology required to meet them is 50% more expensive than the technology required to measure radon accumulations in a slightly less accurate manner. The United States Environmental Protection Agency ("EPA") does not license radon testers. A federal statute, however, authorizes the EPA to advise on the accuracy of various methods of radon testing and to provide to the general public a list of testers that use methods it believes to be reasonably accurate. A recently established company in the state uses a testing method that the EPA has stated is reasonably accurate. The company is also included on the EPA's list of testers using methods it believes to be reasonably accurate. The company applied for a radon testing license in the state, but its application was denied because it cannot demonstrate that its method of testing for radon is sufficiently accurate to meet the rigorous state statutory standards. The company has sued appropriate state officials claiming that the state may not constitutionally exclude the company from performing the required radon tests in the state. Who is likely to prevail in the lawsuit? A) The company, because the Full Faith and Credit Clause of the United States Constitution requires the state to respect and give effect to the EPA's inclusion of the company on its list of testers that use reasonably accurate methods. B) The company, because the Supremacy Clause of the United States Constitution requires the state to respect and give effect to the EPA's inclusion of the company on its list of testers that use reasonably accurate methods. C) The state, because the federal statute and the EPA's inclusion of the company on its list of testers that use reasonably accurate methods are not inconsistent with the more rigorous state licensing requirement, and that requirement is reasonably related to a legitimate public interest. D) The state, because radon exposure is limited to basement areas, which, by their very nature, cannot move in interstate commerce.

C

A city ordinance that gave churches veto power over the grant of liquor licenses for businesses within a quarter mile of the church was struck down by the state's supreme court. In its decision, the court noted that even if the ordinance were not a violation of the federal Establishment Clause, it clearly violated a provision in the state constitution requiring the separation of church and state because it delegated governmental authority to the churches. The supreme court opinion also harshly criticized the city council for passing an ordinance that was clearly a violation of state law. To comply with the court's decision and avoid future criticism, the city council adopted an ordinance prohibiting any cleric from sitting on any public board within the city. A cleric who is currently on a school board within the city brings a suit in federal district court to have the ordinance declared unconstitutional. How should the court rule? A) Uphold the ordinance, because it has a secular purpose and it does not promote excessive entanglement between government and religion. B) Uphold the ordinance, because the previous court decision was based on adequate and independent state grounds. C) Hold the ordinance unconstitutional, because its passage violated the cleric's right to procedural due process. D) Hold the ordinance unconstitutional, because it violates the Free Exercise Clause.

D

A city's water board election laws provide that, although members of the board are elected at large, one member of the board is required to live within each of the five designated water districts within the city. The city's population was more or less evenly distributed among the districts when this election law was enacted. A resident and registered voter of the city investigated the district residency requirement and discovered that most of the city's newer residents had moved into the same two water districts, so that the city's population was no longer evenly distributed among the five water districts. Instead, 80% of the city's residents lived within its central and eastern water districts, while the other 20% of the city's residents were scattered among its three other, more rural, districts. If the resident files suit in federal court challenging the constitutionality of the residency requirement, how will the court most likely rule? A) The residency requirement is unconstitutional because it impairs the voters' equal protection rights, in that it gives the voters in the less populous districts more effective representation on the water board. B) The residency requirement is unconstitutional because it violates the candidates' equal protection rights. C) The residency requirement is constitutionally permissible because the water board members do not exercise legislative power. D) The residency requirement is constitutionally permissible because the water board members are elected at large.

D

A contractor employed by the state was owed $10,000 in back wages by a real estate developer for remodeling work he had completed six months earlier. Pursuant to a state law regulating delinquent payment of wages, the contractor filed an action in state court to resolve the claim within 21 days. In response, the real estate developer filed a voluntary bankruptcy petition in federal bankruptcy court and sought a stay of further proceedings on the unpaid claim. Federal law provided for an automatic stay in both federal and state courts for persons filing federal bankruptcy claims. If the contractor opposes imposition of the stay as applied to his state court claim, what is the proper resolution of the issue? A) The stay cannot be applied in the case, because the state court claim was already filed at the time of the federal bankruptcy petition. B) The stay cannot be applied in the case, because state law requires claims dealing with delinquent payment of wages to be adjudicated by state substantive law. C) The stay can be applied in the case, because Article I gives Congress power to regulate the jurisdiction and procedures of both federal and state courts. D) The stay can be applied in the case, because the Supremacy Clause gives Congress the power to supersede state law with proper legislation.

D

A high school held a graduation ceremony. Attendance at the ceremony is voluntary. The principal of the school is a good friend and follower of a local minister and he convinced the minister to attend the ceremony, say an opening prayer, and congratulate the students for their achievement. The principal instructed the minister to make sure that her opening prayer is nonsectarian, just a general invocation of blessings. The principal even provided the minister with a sample prayer that he believed would pass First Amendment scrutiny. A student who was raised in an atheist home saw the schedule and filed an action to enjoin the school from including the prayer. Should the court grant the injunction? A) No, because the prayer is nonsectarian and, therefore, does not promote any particular religion. B) No, because attendance at graduation ceremonies is voluntary. C) No, because the student is not forced to participate in the prayer and could choose to remain silent. D) Yes, because the prayer violates the Establishment Clause.

D

A large parcel of privately owned land was located next to a river in an area occupied by only a few residences. These homes were largely run-down and had little economic value. The city decided to undertake an economic development plan by using its eminent domain powers to purchase the land from the homeowners, then sell it to a wealthy private development corporation. The developer planned to undertake a comprehensive redevelopment project to benefit the city. The purpose was to create new jobs, increase tax revenues, and revitalize what had been a depressed area of urban blight. One of the homeowners refused to give up her home, despite an offer of compensation from the city. The city instituted condemnation proceedings against the homeowner to take the homeowner's land in exchange for its fair market value. At trial, the homeowner claimed the forced sale was an unconstitutional taking. For which party should the court rule? A) For the homeowner, because the redevelopment plan violated the public use requirement of the Fifth Amendment. B) For the homeowner, because the Takings Clause applies to states and the federal government, not to cities. C) For the city, because the taking is necessary to achieve a compelling government purpose. D) For the city, because taking property from one private party to give it to another private party can be a "public use."

D

A new federal law prohibited the use of various pesticides in areas with a certain population density near navigable waters. A city located in the southeastern United States was plagued by a sharp increase in disease-carrying mosquitoes. The city's board of health recommended that all residential areas be sprayed with a pesticide proven to be highly effective against mosquitoes. Despite the fact that the federal law would prohibit use of that pesticide in these areas, the city council passed an ordinance adopting the board of health plan, relying on the opinions of several independent experts that the health benefits of reducing the mosquito population outweighed the risks of spraying. An environmentally minded citizen of the city brought an action in federal court challenging the ordinance. Assuming that the citizen has standing, is the court likely to find the ordinance valid? A) Yes, because pursuant to the police power, cities have a compelling interest in laws designed to protect the health, safety, and welfare of their citizens. B) Yes, because controlling health hazards is an integral governmental function. C) No, because it is superseded by the power of Congress to adopt laws to protect the health, safety, and welfare of citizens. D) No, because it conflicts with a federal law that Congress had the power to make under the Commerce Clause.

D

A state statute requires that all new automobiles sold in the state shall be equipped with a certain safety system to protect passengers in the event of a collision. An automobile company that wants to sell automobiles in the state files an action to enjoin enforcement of the statute, arguing that the statute deprives the auto company of its right to contract freely with customers under the Due Process Clause. What is the appropriate burden of proof? A) The state must demonstrate a compelling state need. B) The state must demonstrate that the law serves a legitimate government interest. C) The challenger must demonstrate the lack of a compelling state need. D) The challenger must demonstrate the lack of a legitimate purpose.

D

A state wants to open a pilot daycare program for preschool children and, to that end, it submitted the project for open bids as required by state law. The lowest bid that met the criteria established by the state was submitted by a religious organization. The organization's bid is low, in part, because the building in which the program would be housed is tax exempt under state law under a religious institution exemption to property taxes. The organization agreed to operate under the conditions of the project contract, including the provision prohibiting the program from being used to espouse or promote any particular religious view. Studies have shown, however, that children in such situations tend to emulate their caregivers. The state decides to award the contract to the organization. A citizen of the state brings an action in a proper state court seeking to set the award aside, claiming that it violates the United States Constitution. Is the citizen likely to prevail? A) Yes, because the program would constitute an establishment of religion. B) Yes, because the organization's First Amendment rights of freedom of religion and speech would be violated by the contract provision prohibiting it from espousing its religious views. C) No, because the program is necessary to achieve the compelling state interest of educating youth. D) No, because any entanglement with religion is not excessive.

D

A town in a rural state facing financial difficulties passed a variety of "sin taxes," including one aimed at electronic game arcades frequented by local juveniles. The tax is a one cent per game tax imposed on the manufacturers of the games based on the estimated number of plays over a machine's lifetime. There are no electronic game manufacturers in the state. Which of the following constitutional provisions would support the best argument against enforcement of the tax? A) The Equal Protection Clause. B) Substantive due process. C) The Privileges and Immunities Clause of Article IV. D) The Commerce Clause.

D

A township located in a farming community was composed mostly of persons belonging to a specific religious sect. To help instill proper respect for authority in children, which was a central tenet of the sect, and to maintain order in the classroom, the local school board allowed teachers to inflict corporal punishment. Such punishment was inflicted on a fourth grader in a township school immediately after his teacher saw him pulling a girl's hair. Neither he nor his parents belonged to the religious sect. When the boy's parents learned of the incident, they hired an attorney. Rather than suing the teacher for battery as permitted under state law, the attorney brought an action against the teacher under a federal statute providing a cause of action for damages against any government employee who deprives a person of his constitutional rights. Should the court find the policy allowing corporal punishment to be constitutional? A) No, because the punishment policy violates the First Amendment Establishment Clause. B) No, because the boy was denied any kind of hearing, in violation of his right to procedural due process under the Fourteenth Amendment. C) Yes, because under the doctrine of parens patriae states may impose any punishment they see fit. D) Yes, because the punishment was not grossly disproportionate under the Eighth and Fourteenth Amendments.

D

Congress passed a law allowing widespread oil exploration on federal lands in the western United States. A large deposit of oil sand was discovered in one western state and Congress authorized an oil sand refining plant to be built on federal park land within the state. The refinery was built in compliance with federal pollution regulations. Pursuant to state law, the plant manager allowed the state to inspect the plant before putting it into operation. Because state refinery standards were more strict than the federal standards (in order to better protect state citizens from pollution associated with refineries), the refinery did not pass the inspection, and the state inspector refused to give the manager a permit to run the refinery. The refinery manager nevertheless began to run the refinery and was fined by the state. Which of the following is the manager's best defense against imposition of the fine? A) The state does not have a compelling interest in regulating the refinery, because it is within a federal park. B) The state regulation is invalid because Congress has preempted the field of pollution control. C) The state pollution regulation is invalid because it is inconsistent with the state's compelling interest in providing jobs. D) The state law violates the principles of intergovernmental immunity as applied to the manager.

D

The director of a one-person field station of the United States Department of Agriculture ("USDA") in a small town was instructed by his superiors to sell surplus government cheese and butter to local low income residents at 10% of market value. All sales were conducted at the USDA warehouse next to the field station. Pursuant to state statutes allowing municipal governments to establish reasonable regulations governing the retail sale of foodstuffs, the town in which the field office is located requires any establishment for the retail sale of food to pass a health and sanitation inspection and meet other specified criteria for obtaining a city license. The director of the field office failed to obtain a license from the town and was prosecuted for the failure. Which of the following will provide the best defense for the director in this prosecution? A) The ordinance under which the director is being prosecuted is invalid as an undue burden upon interstate commerce. B) The ordinance under which the director is being prosecuted violates the Equal Protection Clause of the Fourteenth Amendment. C) The ordinance under which the director is being prosecuted deprives him of property without due process of law. D) The ordinance under which the director is being prosecuted violates the principles of intergovernmental immunity as applied to him.

D

Thirty years ago, a man obtained a certification from the Federal Housing Administration ("FHA") as a certified real estate appraiser. At the time, few organizations hiring appraisers required certifications. For the past thirty years, the man has worked for State A, appraising properties for tax purposes. He has a contract providing that he can be dismissed only for cause. Six months ago, State A passed a law requiring anyone appraising real estate in State A to hold a State A license as a real estate appraiser. The man, who is 60-years-old and plans to retire in a few years, refuses to spend the time and money necessary to get the State A license. After a hearing, during which the man is given an opportunity to present his case for retaining his position, he is fired, and a much younger employee who has the State A license is hired to replace him. If the man challenges his dismissal in court, who will likely prevail? A) The man, because the state law is preempted. B) The man, because he was deprived of his rights under the Due Process Clause. C) The man, because he was deprived of his rights under the Equal Protection Clause. D) State A.

D

A federal statute authorized the President to designate countries as state sponsors of terrorism and impose economic sanctions on them. When a new President took office, he declared numerous traditional U.S. allies state sponsors of terrorism and imposed sanctions on them. In response, Congress passed a joint resolution, identifying many of the allies as non-sponsors of terrorism and reinstating previously existing trade agreements with those countries. If the President seeks to enforce the sanctions, are they valid? A) Yes, because the joint resolution was not signed by the President. B) Yes, because Congress has the power to regulate domestic commerce, but not foreign commerce. C) No, because Congress has exclusive power to regulate commerce with foreign nations. D) No, because presidential economic power is at its lowest ebb when its use conflicts with the will of Congress.

A

A state has a retirement system under which a fixed percentage of an employee's pay is deducted each payday. Retirement benefits are paid at age 65 for the life of the employee. The state has obtained reliable actuarial statistics that indicate that those who completely abstain from alcohol have a greater life expectancy than those who use alcohol. Those who abstain, therefore, receive smaller monthly retirement benefits than those who use alcohol. A man who has never had a drop of alcohol in his life challenges the lower benefit standard applicable to persons in his class on constitutional grounds. What would be the appropriate burden of proof? A) The man is required to show that the benefit standard is not rationally related to a legitimate state purpose. B) The man is required to show that the benefit standard is not necessary to achieve a compelling state interest. C) The state is required to show that the benefit standard is necessary to achieve a compelling state interest. D) The state is required to show that the benefit standard is rationally related to a legitimate state interest.

A

A city ordinance requires its mayor to have been a resident of the city for at least five years at the time of taking office. A candidate who is thinking about running for mayor in an election that will take place next year will have been a resident for only four and one-half years at the time he would take office. Before he decides whether to run, the candidate wants to know whether he could lawfully assume the position if elected. The candidate filed suit in the local federal district court for a declaratory judgment that the five-year residence requirement is unconstitutional. He named the chairman of his political party as the sole defendant but did not join any election official. The chairman responded by joining the candidate in requesting that the court declare the residence requirement invalid. Which of the following best reflects the action the court should take? A) Refuse to determine the merits of this suit, because there is no case or controversy. B) Refuse to issue the declaratory judgment, because an issue of this kind involving only a local election does not present a substantial federal constitutional question. C) Issue the declaratory judgment, because a residency requirement of this type is a denial of the equal protection of the laws. D) Issue the declaratory judgment, because the candidate will have substantially complied with the residency requirement.

A

A federal act both imposes limits on the amount of sulfur dioxide that can be emitted from electric power plants powered by oil and prescribes that limestone scrubbers be installed on every oil-fueled generator capable of generating more than 10 megawatts of electricity. A state operates two oil-fueled power plants. The first has a capacity of 11 megawatts, but it is used solely to power the state government's facilities in the event of an emergency. The second plant has a capacity of 1,000 megawatts, is operated by the state-owned power authority, and sells electricity both to the state and the general public. Both plants meet the sulfur dioxide emission requirements, but neither has installed limestone scrubbers. The federal entity authorized to enforce the federal act sues the state to enjoin the operation of the two plants until limestone scrubbers are installed. Which of the following is the most likely outcome of the lawsuit? A) The court will enjoin the operation of both plants, because a state must comply with valid federal laws. B) The court will enjoin the operation of only the 1,000-megawatt plant, because the 11-megawatt plant is used by the state in the performance of core state governmental functions. C) The court will not enjoin the operation of either plant, because of the doctrine of intergovernmental immunity. D) The court will not enjoin the operation of either plant, because each plant, by complying with the emission limits, meets the goals of the act, and the state has discretion to decide which means of compliance is most appropriate.

A

A state passed a statute requiring all employers in the state to provide a medical insurance plan for full-time employees. A state trade association to which many state employers belong brought suit in federal court, asking the court to strike down the statute as unconstitutional. Assume that the state statute is not preempted by any federal law. Which of the following best reflects the burden of persuasion in this case? A) The burden is on the trade association to prove that the statute is not rationally related to a legitimate state interest. B) The burden is on the trade association to prove that the statute is not necessary to achieve a compelling state interest. C) The burden is on the state to prove that the statute is rationally related to a legitimate state interest. D) The burden is on the state to prove that the statute is necessary to achieve a compelling state interest.

A

A federal statute enacted about 100 years ago admitted a state to the Union, granted the state certain public lands, and established conditions on the subsequent disposition of these lands by the state. The federal statute also required the state to write those same conditions into its state constitution. One hundred years later, a statute of the state dealing with the sale of these public lands was challenged in a state court lawsuit on the ground that it is inconsistent with the conditions contained in the federal statute and the state constitution. The trial court decision in that case was appealed to the state supreme court. In its opinion, the state supreme court dealt at length with the ambiguous language of the federal statute and with cases interpreting identical language in similar federal statutes. The state supreme court opinion did not discuss the similar provisions of the state constitution, but it did hold that the challenged state statute was invalid, because it was "inconsistent with the language of the federal statute and therefore is inconsistent with the identical provisions of our state constitution." The losing party in the case wishes to appeal the decision to the United States Supreme Court. Can the United States Supreme Court review the decision? A) Yes, the Supreme Court can accept the case for review and determine the validity and interpretation of the federal statute. B) Yes, because the state statute was adopted pursuant to federal law. C) No, on the ground that the decision of the state supreme court rests on an adequate and independent state ground. D) No, because a decision by a state supreme court concerning the proper disposition of state public lands is not reviewable by the United States Supreme Court.

A

A federal statute imposed criminal penalties for killing certain specified animals that have been determined by Congress to be of importance to the tourism industry in the region in which the animal was located. Among the animals protected was the red fox. However, a state in which the red fox was located classified the animal as a varmint that could be destroyed at will by anyone with a general hunting license. A rancher who possessed a valid state general hunting license regularly shot and killed red foxes that ate his artichoke plants. If the rancher is prosecuted under the federal statute and challenges the constitutionality of the law, which of the following is the strongest constitutional argument in support of the federal statute? A) The commerce power. B) The Necessary and Proper Clause. C) The police power. D) The power to regulate federal lands.

A

A federal statute just signed into law by the President provided that school districts no longer needed to recognize the tenure of elementary school teachers—all tenured teachers would lose their status and would be treated the same as nontenured teachers. The effect of the law would be to allow all tenured teachers to be fired more easily if their performance was not adequate. The law also allowed the salaries of tenured teachers to be lowered, at least until a new contract with the teachers could be negotiated. The law had a two-year grace period before it was to take effect, to give schools and teachers time to adjust to the law; however, it specifically provided that once it is in effect, school board actions under the law supersede any existing contract terms. A public elementary school district is in the first year of a three-year union contract with its teachers. The school board has stated that it plans to abolish tenured positions as soon as the law takes effect. The union, believing that numerous terms of the contract will be invalidated when the law takes effect, filed an action in federal court on behalf of the teachers, asking for an injunction to prevent the school board from abolishing tenured positions and for a declaratory judgment stating that the law is invalid. Should the federal court hear the case? A) No, because a ruling on the law at this point is premature. B) No, because the union does not have standing to sue on behalf of the teachers. C) Yes, because the federal law encourages improper interference with a contract in violation of the Contract Clause of the Constitution. D) Yes, because the teachers' rights and benefits are threatened by the law and the school board's stated plans.

A

A foreigner in the United States attempted to kill the head of his country while the leader was confined to a state's hospital. The foreigner was tried in the state for the crime of attempted murder, and he was sentenced to 10 years in prison. Two years later, the President wanted to improve diplomatic relations with the foreigner's country, so the President pardoned the foreigner and ordered his release. Which of the following presents the strongest argument available to the governor of the state in declining to release the prisoner? A) The presidential pardon power does not extend to the foreigner. B) The President's power as Commander in Chief does not extend to a state's prisoners. C) When the governor of a state acts pursuant to the state's constitution, he is exempt from regulation by the federal government. D) The President, under the constitutional mandate to execute the laws, must honor any valid judgments of a state's courts.

A

A man committed a particularly brutal series of crimes that, because of their interstate character, were violations of a federal criminal statute. The man was convicted in federal court and sentenced to life imprisonment. Six months after the man was incarcerated, the President pardoned him. There was a great public outcry, amid charges that the President issued the pardon because the man's uncle had made a large contribution to the President's reelection campaign fund. Responding to public opinion, Congress passed a bill limiting the President's power to pardon persons convicted under the specific statute that the man had violated. The President vetoed the bill, but three-quarters of the members of each house voted to override the veto. Is the legislation constitutional? A) No, because the power to pardon for federal crimes is expressly granted to the President in the Constitution and is an unqualified power (except as to impeachment). B) No, because the President has the duty to enforce the laws, and therefore has plenary powers. C) Yes, under Article I, Section 1. D) Yes, because Congress wrote the federal criminal statutes and has the right to determine who should be convicted under such statutes.

A

A state legislature enacted an excise tax on any automobile sold in the state that had not been manufactured within the state. The tax was intended to ease the desperate plight of the thousands of auto workers suffering layoffs, plant closures, and pay cuts from lost sales to foreign competitors. The tax was graduated, from 5% of the sales price of inexpensive automobiles down to 1% for automobiles selling for more than $100,000. A corporation that manufactures automobiles in a neighboring state brought an appropriate action in federal court to enjoin enforcement of the automobile tax statute as to its products. Which of the following is the strongest constitutional argument supporting the invalidity of the special tax? A) It is an undue burden on interstate commerce. B) It violates the Equal Protection Clause of the Fourteenth Amendment. C) It violates the Fourteenth Amendment's protection of the privileges and immunities of national citizenship. D) It violates the Due Process Clause of the Fourteenth Amendment.

A

Which of the following suits would not fall within the United States Supreme Court's original jurisdiction under Article III, Section 2? A) A suit seeking to assert the interest of state citizens in retaining diplomatic relations with a foreign nation. B) A suit seeking to protect a state's timber from allegedly illegal cutting by residents of another state. C) A suit seeking to enjoin enforcement of an allegedly unconstitutional executive order that will greatly limit the state's authority to make policy decisions regarding admission to state universities. D) A suit by the United States Government seeking to enjoin state construction of a bridge over a navigable waterway.

A

Scientific studies indicate that emissions from gasoline combustion engines contribute to a reduction in the health and growth rate of trees. To protect trees in the national forests, the Environmental Protection Agency ("EPA"), a federal agency, enacted stringent new fuel emission standards that apply to motor vehicles operating inside or within a 10-mile radius of all federal parks. The EPA regulations expressly preempt any conflicting state or local regulations, unless the state or local regulations require an even greater reduction in automobile emissions. A shuttle service operator transports tourists from hotels to the front entrance of a federal park. If the EPA regulation applies to her shuttle service, the operator will have to spend several thousand dollars for each shuttle to modify the engine exhaust system. The operator brings suit to challenge the validity of the federal regulation as applied to her. How will the court likely rule on the shuttle operator's challenge? A) Find the statute constitutional, as a proper exercise of Congress's power to regulate federal property. B) Find the statute constitutional, as a proper exercise of Congress's power to regulate for the general welfare. C) Find the statute unconstitutional, because Congress cannot regulate activities occurring outside a federal park. D) Find the statute unconstitutional, because this is an area reserved to the states under the Tenth Amendment.

A

Which of the following Answers is most accurate regarding the "original" Constitution, including the first 10 amendments (Bill of Rights)? A) The Constitution was primarily concerned with the structure of the federal government, common defense, and eliminating barriers to trade. B) The Constitution strongly limited the power of states to regulate their own citizens C) The Constitution gave the Federal Government full power to do anything it wanted to US citizens, including enslaving or exterminating them. D) The Constitution allowed the states to have exclusive control over certain subject areas such as education.

A

Which of the following acts would be improper for the United States Senate to perform? A) Adjudicating a border dispute between states. B) Defining certain qualifications for being a member in good standing of the United States Senate. C) Sitting in joint session with the House of Representatives. D) Passing a resolution directing the President to pursue a particular course of foreign policy.

A

Which of the following statements is the BEST answer regarding justiciability? A) In order for a case to be justiciable, there must be, at a minimum, at least two parties who are sufficiently invested in the outcome of the case to fully present the issues involved in their case for adjudication. B) If a case is not sufficiently ripe, it is usually moot. C) A case is always justiciable when at least one of the parties has suffered real, demonstrable damages to their legal interests. D) In LA v. Lyons, the chokehold case, the Court determined that the case was not justiciable because the issue presented was a political question.

A

Which of the following statements is the BEST answer? A) Congress cannot generally reach private discrimination under the 14th Amendment. B) Congressional laws banning discrimination in restaurants/hotels were based on the 14th Amendment's Equal Protection clause. C) The Supreme Court vigorously upheld the enforcement of the Civil Rights Acts and Congressional laws based on them. D) The Commerce Clause has been the only basis for Congress to prohibit private discrimination or racist acts

A

Which of the following statements is the BEST answer? A) Despite the ruling in Glucksberg, states may allow physician assisted suicide. B) States may not allow physician assisted suicide. C) The Substantive Due Process Right to Privacy guarantees the right to physician assisted suicide. D) Because of the ruling in Glucksberg, palliative care is not permitted in any state.

A

Which of the following statements is the BEST answer? A) The Court in the "Lochner Era" used the 14th Amendment to limit states' ability to regulate the workplace and a narrowed Commerce Clause to limit the Federal Government's power to regulate. B) The Court in the Lochner Era used the Substantive Due Process Clause of Article V to limit states' ability to regulate the workplace. C) In the "Lochner Era" the court used the Substantive Due Process to enable state legislation to protect workers. D) The "Lochner Era" approach to the Substantive Due Process was largely responsible for the growing protection of workers' rights following the Great Depression.

A

Which of the following statements is the BEST answer? ( Single Choice) A) The Supreme Court, on at least one occasion, has ruled that "literacy tests" do not violate the 14th or 15th Amendments, even though they have usually been used to prevent African-Americans from voting. B) The Supreme Court recently upheld that portion of the Civil Rights Voting Act that prevented certain states with a history of discrimination from changing voting procedures without pre-clearance from the Attorney General. C) The Court in Shelby County held that deference to Congress required that Section 4 of the Voting Rights Act had to be upheld because it was almost unanimously renewed in bipartisan votes in Congress. D) The Supreme Court has struck down a Congressional Act that banned literacy tests in voting.

A

A special prosecutor was appointed to investigate allegations that a presidential aide accepted bribes to secure the appointment of certain federal judges. The special prosecutor has subpoenaed all memoranda written by the aide on the subject of judicial nominations to vacancies. The President has government attorneys move to quash the subpoena on the ground of executive privilege. How should the trial judge rule on the motion? A) Deny the motion, because the President has no greater privilege in this matter than any other officer of the government. B) Deny the motion, because although there is a presumption of a privilege, it can be overcome by a strong showing of the needs of the criminal justice system. C) Grant the motion, because executive privilege is an absolute bar to the requested information, except in the case of an impeachment of the President. D) Grant the motion, because executive privilege is an absolute bar to the requested information, as the President is entitled to confidential advice from subordinates.

B

As part of his official duties, a chief of a state's department of employment services is expected to set forth guidelines for the payment of unemployment benefits to the state's unemployed citizens. Because of a recession and high unemployment, the state receives money from the federal government to extend the benefit period for those citizens who have exhausted their usual employment insurance benefits. The chief, however, decided that there are sufficient employment opportunities and terminated the benefits of those persons who were receiving payments from the federal funds. The citizens of the state affected by this administrative decision brought suit in federal district court against the chief for reimbursement of unpaid benefits and declaratory relief prohibiting the chief from withholding funds in the future. After a trial on the merits, the federal district court ordered that the citizens be reimbursed their lost unemployment benefits and that the chief disburse the emergency funds in the future in accordance with federal law and guidelines. The chief appealed to the federal circuit court which upheld the lower court's decision. How is the Supreme Court likely to rule on the appeal? A) Reverse the decision, because the suit is barred by the Eleventh Amendment. B) Reverse the monetary reimbursement, but uphold the declaratory relief. C) Uphold the decision, because the suit is not against the state, but against the chief personally, even though he was acting in his official capacity. D) Uphold the decision, because by accepting the federal funds, the state has waived its immunity from suit.

B

Congress created the National Agency for Burglar Alarms ("NABA"), giving it the power to regulate both burglar alarm hardware and installation personnel. NABA adopted a regulation requiring that all burglar alarm installation companies be licensed, and providing that anyone installing an alarm without a license could be fined. The regulation also provided that any company in the installation business on the day the regulation was adopted automatically would receive a license, but to obtain a license thereafter, an applicant would have to show that he has worked as an installer at a licensed company for at least three years. A man who has been installing alarm systems for eight years sold his installation business a few months before the NABA regulation was adopted and went to work for the purchaser servicing his old accounts. A few months after the NABA regulation was adopted, a representative from a national department store chain approached the man with an offer to hire him as an independent contractor to revamp the chain's alarm systems. The man quit his job and applied for a NABA installer's license. His application was denied because he was not in business on the day the NABA regulation was adopted and had worked for a licensed installer for only a few months. The man decided to install the alarm systems anyway. Can the man properly be fined for installing alarms? A) Yes, because the NABA was established under Congress's power to legislate for the general welfare, and Congress may take whatever steps are necessary and proper to enforce its laws. B) Yes, because the regulation falls within the scope of Congress's commerce power, and Congress may delegate its authority to regulate as it has done here. C) No, because the regulation interferes with the man's fundamental right to earn a living without a substantial justification and so violates the Privileges and Immunities Clause of Article IV, Section 2. D) No, because a government agency cannot itself levy fines for a violation of its regulations.

B

Congress has the following powers: A) To enact any legislation that is Necessary and Proper for the General Welfare of the United States. B) To enact any legislation that is Necessary and Proper for the regulation of commerce. C) To enact any legislation that is in the General Welfare of the United States. D) To pass any law that is reasonably appropriate for the proper governing of the country.

B

Congress passed a statute requiring that all beekeepers be licensed and establishing standards for the maintenance of the hives, the removal of honey, and the labelling of honey for sale. Which of the following provides the strongest basis for enactment of this legislation by Congress? A) The General Welfare Clause. B) The Commerce Clause. C) The Supremacy Clause. D) Congress's police power.

B

Congress wishes to enact legislation prohibiting discrimination in the sale or rental of housing on the basis of the sexual orientation of the potential purchaser or renter. Congress wants this statute to apply to all public and private vendors and lessors of residential property in this country, with a few narrowly drawn exceptions. Which of the following provides the strongest basis for enactment of this legislation by Congress? A) The General Welfare Clause, because the conduct the statute prohibits could reasonably be deemed to be harmful to the national interest. B) The Commerce Clause, because the sale or rental of almost all housing in aggregate could reasonably be deemed to have a substantial effect on interstate commerce. C) The enabling clause of the Thirteenth Amendment, because that amendment prohibits discrimination against badges of past discrimination. D) The enabling clause of the Fourteenth Amendment, because that amendment prohibits public and private actors from engaging in discrimination.

B

A city that lies astride a major interstate highway recently passed a referendum declaring itself a "nuclear-free zone." The referendum included a provision making criminal any importation of specified nuclear materials into the city limits. The law was immediately challenged in federal court by an interstate trucking firm that regularly transported prohibited nuclear materials through the city on the highway. The case ultimately reached the United States Supreme Court, which held that the challenged ordinance was constitutional because the city had a rational basis for concluding that the citizens of the city would be safer if the prohibited materials were kept outside of town, and because the ordinance did not unduly burden interstate commerce. Many other towns and cities throughout the nation considered similar enactments after the Supreme Court decision was announced. In response, Congress enacted a federal statute prohibiting the state regulation of interstate transportation of nuclear materials. If the statute is challenged in federal court, is it likely to be held constitutional? A) No, because the Supreme Court has already ruled that local governments may prohibit specified nuclear materials from crossing their borders. B) No, because the disparate treatment of interstate versus intrastate carriers of nuclear materials would violate the Equal Protection Clause of the Fourteenth Amendment. C) Yes, because Congress has very broad power to regulate interstate commerce. D) Yes, because the Supremacy Clause requires that state enactments bow to conflicting federal legislation.

C

A group of ex-convicts have come to you in the State of Novania and have asked you to litigate a challenge to the state's ban on voting for ex-felons. The lawsuit is being brought in federal court. What are the best arguments you could make on their behalf. A) Discrimination in voting against ex-felons should be prohibited under the Substantive Due Process Clause. B) Ex-felons constitute a suspect class because the legislation denying them the right to vote is based on animus and thus is not a legitimate governmental objective. C) The right to vote for all US citizens is guaranteed by the US Constitution. D) Discrimination against ex-felons constitutes discrimination against a group with respect to a fundamental right.

C

A hairdresser owns a beauty salon catering to female customers. The hairdresser employs only male hairdressers at her salon. A city ordinance makes it unlawful for any person to operate a beauty salon if the hairdressers provide services to clients of the opposite sex. The hairdresser brings an action in federal court challenging the constitutionality of the ordinance. If the city moves to dismiss the lawsuit, arguing that the hairdresser lacks standing, is the city likely to prevail? A) Yes, because the ordinance does not prohibit the operation of beauty salons per se, only the right of hairdressers to service customers of the opposite sex. B) Yes, because only the employees can raise their rights of association. C) No, because the hairdresser has rights that are harmed by the ordinance. D) No, because the city cannot prove the ordinance serves a substantial governmental interest.

C

A husband and wife were both professors at a federal military academy. The husband made a speech criticizing United States foreign policy with respect to a foreign country and was dismissed from his teaching position soon thereafter. Six months later, he accepted new employment in another state. The wife then commenced suit in federal court, claiming that the academy violated her husband's right to due process and his right of free speech when it fired him. If the court dismisses the suit, which of the following presents the most likely ground for the dismissal? A) The political question doctrine, because foreign policy is in the domain of the executive branch. B) Mootness, because the husband has a new job. C) The plaintiff's lack of standing. D) The lack of a federal question.

C

A man had been a lawful resident alien for several years. He was browsing an online website for government employment opportunities and saw an opening for a position as a state highway patrol officer. He decided to apply for the position and was granted an interview. At the interview, the man was informed that although his application had complied with all the requirements necessary for the position, he was disqualified from service solely because he was not a U.S. citizen. The man brought an action for declaratory relief, claiming that the state has violated his rights under the Fourteenth Amendment. Should the man be successful in his action? A) Yes, because the state has not shown the classification restricting aliens is necessary to achieve a compelling interest. B) Yes, because the man's application complied with all the requirements necessary for the position. C) No, because serving as a highway patrol officer is integral to the self-government process. D) No, because the state may discriminate against aliens where the justification is rationally related to a legitimate interest.

C

A state imposed a tax on the "income" of each of its residents. As defined in the taxing statute, "income" includes the fair rental value of any automobile provided by the taxpayer's employer for the taxpayer's personal use. The federal government supplies automobiles to some of its employees, who are residents of the state. The federal government allows the employees to use the automobiles for both official and personal use. There is no federal legislation on this subject. May the state collect the tax from the federal employees on the fair rental value of the personal use of the automobiles furnished by the federal government? A) No, because a tax on the federal employees is essentially a tax on the United States. B) No, because the tax would be a tax on federal property as the automobiles are owned by the federal government and primarily used by federal employees in the discharge of their official duties. C) Yes, because the tax is imposed on the employees, rather than on the United States, and the tax does not discriminate against persons who are employed by the United States. D) Yes, because an exemption from such state taxes for federal employees would be a denial to others of the equal protection of the laws.

C

Which of the following statements is the BEST answer regarding the Constitution (pre-Civil War) and slavery? A) The Constitution allowed individual states to free slaves who escaped to the non-slave states.. B) Under the Constitution only individuals of African descent could be enslaved and it protected against people being enslaved by their state on the basis of their religion. C) The Constitution prohibited the federal government or the states from ending the importation of slaves before 1808. D) The Constitution prevented the slave trade upon ratification, although it allowed slavery to continue in the states that allowed it.

C

A state law required all automobile drivers to carry liability insurance; however, because of the high number of auto accidents in the state, the cost of insurance became prohibitive. A study sponsored by the state legislature showed that males under the age of 21 were four times more likely to get into automobile accidents than any other group, including females in the same age group. The study predicted that prohibiting males under the age of 21 from driving would result in a 15% reduction in all other persons' automobile insurance rates. Ultimately, the legislature raised the minimum age for obtaining a driver's license to age 21 for males. Females were still allowed to obtain licenses at age 16. An 18-year-old male living in the state when the limit was raised, and who worked as a pizza delivery driver, was fired from his job and replaced by a 17-year-old female. If the young man sues to have the law set aside and prevails, what is the most likely reason? A) The state could not prove that the law was the least restrictive means of achieving a compelling government purpose. B) The state could not prove that the law was rationally related to a legitimate government purpose. C) The state could not prove that the law was substantially related to an important government interest. D) The state could not prove that the law was necessary to achieve a compelling government purpose.

C

A state legislature passed a statute denying in-state tuition rates at any state-supported college or university for any resident aliens who were citizens of "any country which, during the previous five-year period, has held citizens of the United States as hostages." Before becoming a resident alien of the state three years ago, the plaintiff had been a citizen of a country that had held United States citizens as hostages. Prior to passage of the statute, he had been enrolled for two years at the state university and had qualified for the in-state tuition rates because he was a resident of the state. The registrar of the university has informed the plaintiff that he no longer qualifies for the in-state tuition rate, and he cannot afford to pay the nonresident tuition. If the plaintiff files an appropriate suit asking the federal courts to strike down the state statute, which of the following is his best argument? A) Because higher education is now so important for earning a livelihood, it has become a fundamental right protected by the Privileges or Immunities Clause of the Fourteenth Amendment. B) Resident aliens cannot be blamed in retrospect for actions of their governments and, therefore, the state statute violates the constitutional prohibition of ex post facto laws. C) Alienage is a suspect category under the Equal Protection Clause of the Fourteenth Amendment, and the state has not met the appropriate standard to burden the plaintiff. D) Attending a state-supported university in a state where one is a legal resident is a property right that may not be taken away without due process of law.

C

A state provided for a public school system based primarily on property tax revenues from the various districts. School districts that had a property tax base below a certain threshold received supplemental funds from the state that were derived from state lottery revenues. The school districts receiving the supplemental funds served a predominantly Hispanic population as compared to the school districts funded only from property tax revenues. To help balance its budget this year, the state legislature passed a statute terminating the supplemental funds program and earmarking the lottery revenues for deficit reduction. A group of parents of Hispanic schoolchildren in one of the school districts formerly receiving supplemental funds filed suit in federal court, alleging that the state's action in terminating the funding violates the Equal Protection Clause of the Fourteenth Amendment. Which of the following best describes the appropriate standard by which the court should review the constitutionality of the state action? A) Because the state statute results in discrimination against a suspect class, the state will have to demonstrate that the statute is necessary to vindicate a compelling state interest. B) Because the right to education burdened by the statute is not a fundamental right, the parents will have to demonstrate that the statute is not substantially related to an important state interest. C) Because no suspect class or fundamental right is improperly burdened in this case, the parents will have to demonstrate that the statute is not rationally related to any legitimate state interest. D) Because the state statute is not discriminatory in intent, the state will have to demonstrate only that the statute is rationally related to a legitimate state interest.

C

A state public employee retirement act provided that, while legitimate children under 18 qualify for survivor benefits, an employee's children born out of wedlock may recover only if they lived with the employee in a regular parent-child relationship. A state employee lived with a woman in the state for 10 years, after which they separated. They had two children, both of whom were the employee's natural children born out of wedlock. The employee supported the children under a state child support decree until he died a year ago. At the time of his death, he was covered by the retirement act. The state retirement commission determined that the children did not qualify because they were living with their mother and not living with the employee at the time of his death. The mother sued in federal court alleging that, if the children were born in wedlock, they would have been entitled to benefits, and that it was discriminatory to treat illegitimate children differently. Is the state provision constitutional? A) Yes, because a state may allocate survivor benefits to its employees without restriction. B) Yes, if there was a rational basis for the classification of illegitimate versus legitimate children. C) No, unless the classification of legitimate and illegitimate children is substantially related to an important state purpose. D) No, unless the different treatment of legitimate and illegitimate children is necessary to promote a compelling state interest.

C

A state recently enacted a statute making it illegal for "anyone to cause any substance that is likely to harm fish or aquatic plants" to be discharged into any body of water in the state. The statute was aimed at curtailing the dumping of industrial waste into the state's waters. The state attorney general has sole authority to enforce the statute and may seek either fines or injunctive relief. Farmers in the state use fertilizer on their crops, which could be carried by rain into lakes and streams and might harm aquatic life. The farmers fear that the statute will be applied to them. When the statute became effective, three local farmers brought an action in federal district court, seeking a declaratory judgment that the statute is unconstitutionally vague and injunctive relief against its enforcement because enforcement against the farmers would destroy their livelihood. Which of the following doctrines would be most likely to prevent the case from being heard on its merits? A) Mootness, because the state has not enforced the law against the farmers. B) Standing, because the three farmers cannot properly represent all state farmers. C) Abstention, because there is an unsettled question of state law. D) Political question, because this issue should be decided by the state legislature.

C

A state statute imposes limits on the amount of sulfur dioxide that can be emitted from electric power plants powered by oil and prescribes that limestone scrubbers be installed on every oil-fueled generator capable of generating more than 10 megawatts of electricity. There is no contradictory federal statute on this matter. The federal government owns an electric power plant powered by oil in the state. The plant is used solely to power the local federal government facilities and is operated by a private corporation under a contract with the federal government. The plant has a capacity of 11 megawatts, but it meets the sulfur dioxide emissions requirements. It has not installed limestone scrubbers. The state prosecutes the private operator of the plant for violation of the state statute. Is the operator likely to be convicted? A) Yes, because the regulation of pollution is a legitimate state police power concern. B) Yes, because the regulation of pollution is a joint concern of the federal government and the state and, therefore, both may regulate conduct that causes pollution. C) No, because the operations of the federal government are immune from state regulation in the absence of federal consent. D) No, because the power plant is not in substantial violation of the state pollution control standards.

C

A state statute required all persons operating a catering business within the state to be licensed by the state board of health. The statute further provided that licenses will be granted only to applicants who have resided in the state for five years and who are citizens of the United States. A long-time resident of the state who was not a United States citizen challenges the requirement that applicants for licenses must be citizens of the United States. This provision is most likely: A) Constitutional, as an exercise of the state's police power. B) Constitutional, as an effort to protect the health and welfare of the residents of the state. C) Unconstitutional, as a denial of equal protection. D) Unconstitutional, as a bill of attainder.

C

An act of Congress provides for the payment of Social Security benefits to the disabled children of deceased workers. The benefits are payable even after the child reaches the age of majority, but terminate automatically if the recipient marries, unless the recipient marries a person who is also disabled, in which case the Social Security payments continue. The plaintiff has been disabled since birth, and her father died when she was 14 years of age. She has received Social Security benefits since her father's death. However, at age 25, the plaintiff married. Her husband is not disabled, and thus her Social Security benefits were summarily terminated. She files suit in federal district court, seeking to compel the government to restore her benefits. She asserts, among other things, that there is no logical reason why she should be deprived of benefits when unmarried disabled persons and disabled persons who marry other disabled persons continue to receive them. Which of the following provides the strongest constitutional basis for the plaintiff's suit? A) The privileges and immunities of national citizenship protected by the Fourteenth Amendment. B) The Equal Protection Clause of the Fourteenth Amendment. C) The Due Process Clause of the Fifth Amendment. D) The prohibitions against bills of attainder.

C

In compliance with a federal statute that permits government agencies to sell or give away surplus government property, the Secretary of State directed that one of the State Department's surplus airplanes be given to a church. The Secretary knew that the church planned to use the plane to fly medical supplies to its missions in Third World countries. These missions provide medical assistance, but they also attempt to evangelize residents of the countries in question, and the Secretary was aware that, in addition to medical supplies, the plane might transport Bibles and religious tracts translated into local languages. Had the Secretary not ordered the plane to be given to the church, it would have been sold at a very reasonable cost to a nonprofit organization that helps teach young people the fundamentals of piloting and maintaining aircraft. Which of the following parties would be most likely to have standing to sue to prevent the Secretary of State from making the gift to the church? A) A taxpayer. B) A citizen of the United States. C) A member of the nonprofit flying organization. D) The attorney general of the state in which the airplane is located.

C

In exchange for an agreement by a foreign nation to stop all testing and development of its nuclear arsenal, Congress passed a bill authorizing $100 million to enable students from that country to attend colleges and universities in the United States to obtain higher education. Two weeks after the measure was passed, human rights violations were committed in the foreign nation, during which a government-sponsored coalition used chemical weapons to kill hundreds of women and children. Immediately, the President ordered troops to be sent to the foreign country and he canceled the appropriation of the educational funds. Is the President's action to cancel the higher education measure constitutional? A) Yes, because the President has exclusive power over the area of foreign affairs. B) Yes, because of the President's broad emergency powers as commander-in-chief of the military. C) No, because of the President's obligation to take care that the laws are faithfully executed. D) No, because the higher education measure derives from Congress' power to regulate commerce with foreign nations.

C

Police departments of bordering states have been cooperating in an investigation of a man who appeared to be selling illegal drugs. They had evidence that the man would receive drugs at an office he rented in State A and sell them out of an apartment he rented in State B. He would then deposit the proceeds of the sales into a bank located in State A. Once enough evidence was gathered, the man was arrested by police officers in State A. Each state has a forfeiture statute allowing the state to seize proceeds of drug sales. State A seized the proceeds of the man's bank account. State B requested a share of the proceeds, but its request was denied. Officials in State B file suit in the United States Supreme Court, demanding a share of the drug proceeds from the banking state. Does the United States Supreme Court have jurisdiction to decide this issue? A) No, because the suit must be brought in state court and pursued through the usual appellate channels before a decision of the highest state court may be appealed to the United States Supreme Court. B) No, unless both states agree. C) Yes, even if a state is named as the party-defendant. D) Yes, as long as the suit is not brought against State A but rather against a State A official to prevent that official from taking some action.

C

The United States Surgeon General was cited for contempt for refusing to answer questions as part of a Senate investigation regarding an issue in the Food and Drug Administration. His contempt citation will be dismissed if he can show which of the following? A) As a member of the executive branch, he is immune from prosecution. B) If he answered the questions, he could be subject to dismissal from his position as Surgeon General. C) The questions do not relate to any matter concerning which the Senate may legislate. D) The questions do not relate to any matter concerning current or planned legislation.

C

The Vice President conspired with various Pentagon and CIA officials to sell military weapons to a foreign country. When the President learned of these activities, he launched a full-scale investigation into the matter. The President presented his findings to Congress. The House of Representatives, by a one-vote majority, voted to impeach the Vice President. The Senate, by a one-vote majority, voted to remove the Vice President from office. If the Vice President refuses to leave office, what is her best argument for retaining her position? A) Only the President, not the Vice President, may be impeached. B) A two-thirds vote of both Houses of Congress is required to convict. C) The Senate did not have enough votes to convict her. D) The Senate, not the House of Representatives, is the appropriate body to impeach an executive officer.

C

To combat rising unemployment, a state offered a $25,000 prize to anyone who could devise a scheme to create at least 200 jobs within the state and demonstrate its viability. While hiking through a national park within the state, a geologist noticed rock containing titanium. Knowing that titanium was commonly used in military aircraft built within the state and that mining and refining titanium could provide the state with thousands of jobs, the geologist chipped out a sample of the ore and took it back to the state employment division. After reviewing the geologist's ideas, the state announced in a press release that he was the first recipient of the $25,000 prize. Within a few days, the federal ranger in charge of the valley from which the sample was taken had the geologist arrested for violating a federal law making it illegal to remove any "plants, animals, or minerals from federal lands." The geologist was convicted and fined $5,000. He appeals the conviction to the federal court of appeals, claiming that the fine is unconstitutional. How should the court rule? A) For the geologist, because the state has a compelling interest in reducing unemployment and the federal statute unreasonably interferes with the state interest. B) For the geologist, because removing the ore was a purely intrastate act and had no effect on interstate commerce. C) For the government, because the federal statute providing for the fine is constitutional under the Property Clause of Article IV, Section 3 of the federal Constitution. D) For the government, because the federal statute providing for the fine is constitutional under the Commerce Clause.

C

To combat the problem of obesity among the nation's youth, a state adopted a statute providing funds to train a corps of nutritionists who could go to schools and run six-week seminars designed to encourage healthy eating and exercise habits. The administration of a private school that has a stated policy of admitting only white students has requested that a state nutrition corps member be sent to the school to run a six-week seminar. Which of the following is the best constitutional argument supporting the state's rejection of the request? A) The Constitution prohibits private discrimination. B) The state may not aid private schools. C) Segregation would be furthered by the provision of the seminar. D) No legitimate educational function is served by giving nutrition seminars at private schools.

C

Which of the following statements is the BEST answer regarding State Action? A) In Shelley v. Kraemer (1948), the Supreme Court held that a racially restrictive covenant in a land deed in a sale between two private individuals could be enforced because there was no state action and therefore no 14th Amendment violation. B) In US v. Morrison, the Supreme Court held that the Violence against Women Act could be sustained as a federal remedy for state violations of the 14th Amendment. C) State inaction can constitute state action. D) Under current law, speech cannot be restricted in shopping centers because shopping centers are analogous to town halls.

C

Which of the following statements is the BEST answer? A) Equal Protection is applied against the states through the Due Process Clause of Article V and is applied against the federal government through the Due Process Clause of the 14th Amendment. B) Equal Protection is applied against the states through the 14th Amendment and is applied against the Federal Government through the Due Process Clause of Article I. C) Substantive Due Process under the 5th and 14th Amendments is the process by which the state make take away your right to "life, liberty or property." D) Substantive Due Process defines what is meant by "life, liberty and property."

D

A federal statute provides for subsidized health care benefits for all full-time college students who are also unemployed. The benefits terminate automatically if the recipient fails to enroll in college full-time in any two consecutive semesters or takes a job working more than 15 hours per week. A college student who had previously received the benefits was denied coverage when he reported that he had taken a full-time student teaching job his senior year, a requirement for his degree. The student did not receive health care benefits through his employer and was left with no coverage. The student filed suit in federal court to compel the government to continue providing him with the subsidized benefits while he finished his degree. He asserts, among other things, that there is no logical reason why he should be deprived of benefits when students in other fields who are not required to work as part of their degree program continue to receive them. Which of the following is the strongest ground for the government to defend the constitutionality of the federal statute in question? A) Congress's power over federal expenditures is plenary, and Congress has discretion to spend money from the federal treasury in whatever way it wishes. B) Because there is no constitutional entitlement to health care benefits, Congress is free to condition them as it sees fit. C) The classification imposed by this statute is substantially related to the important government interest of saving money. D) Congress could rationally conclude that an unemployed college student is less likely to be covered under an employer's coverage and thus more needy than an employed student.

D

A man was arrested in a state for armed robbery. A combined preliminary hearing to determine probable cause and initial appearance was held within 20 hours of his arrest. Probable cause was found, and bail was properly denied under the state's Bail Reform Act. A state statute provided that when a defendant is in custody, his trial must begin within 50 days of his arrest. After 50 days had passed since the man's arrest and no trial had been held, he filed a motion for dismissal for violation of his right to a speedy trial under the state constitution, which tracked verbatim the speedy trial provision of the United States Constitution. The trial judge held that he was bound to follow federal interpretations of the speedy trial provision and granted the man's motion on that basis. On appeal, the state supreme court agreed with the trial judge. The state prosecutor seeks to challenge the ruling in the United States Supreme Court. If the Supreme Court thinks that the state court wrongly decided that the man was denied his right to a speedy trial under federal standards, how should it proceed? A) Reverse the decision because the state speedy trial provision cannot be interpreted in a manner different from federal interpretations. B) Reverse the decision and remand it to state court because the state speedy trial issue was so intertwined with the federal question that it would be difficult to determine on which ground the state court relied. C) Decline jurisdiction because the Eleventh Amendment prohibits a state from challenging a decision of its supreme court in federal court. D) Reverse the decision and remand the case to be decided on the independent state grounds only.

D

Following a series of terrorist attacks in a small foreign country in which five American citizens were taken hostage, the President issued an executive order to cut off all diplomatic ties with that nation, including a prohibition on virtually all trade agreements. Under the terms of the executive order, the country's embassy was closed and its consul was ordered to leave the United States. The Prime Minister of the foreign country, which imports two billion dollars of corn from the United States each year, threatened to stop all U.S. imports immediately unless the embassy was reopened and the consul reinstated. The consul brought an action against the United States in federal district court to enjoin enforcement of the President's action. Will the court hear the case? A) Yes, because the President's action usurps the power of Congress to regulate issues directly or indirectly affecting foreign commerce. B) Yes, because the consul has suffered redressable harm. C) No, because lower federal courts lack original jurisdiction over cases involving ambassadors and consuls. D) No, because Article II gives the President exclusive authority to recognize foreign governments.

D

A state that is subject to severe winters generally allows the use of studded tires between October 1 and March 31. However, the legislation allows counties to opt out and prohibit the use of studded tires year round, because studded tires tend to tear up pavement more than non-studded tires, thus necessitating more frequent road repairs. No other state in the region allows use of studded snow tires at all. The state law contains one exception: it excludes "doctors" from any county ban on the use of snow tires because they might have to cross county lines in emergencies. After the passage of the legislation, only one county in the state invoked its right to ban the use of studded snow tires. A lawyer who lives in the state was angered that the legislature had given special privileges to doctors but not to lawyers. One January day, with studded tires on his car, he drove from his home county, which allowed use of studded tires, into the county that banned them. A sheriff's officer noticed the lawyer's studded tires and cited him. After being convicted and fined, the lawyer appealed. What is the lawyer's best argument for getting the ban invalidated? A) The statute interferes with his fundamental right to practice his profession in violation of the Privileges and Immunities Clause of Article IV. B) The statute violates his right to travel. C) The statute violates the Commerce Clause by placing an unreasonable restraint on interstate commerce. D) The ban on studded snow tires is not rationally related to a legitimate state interest because it will likely result in an increased loss of life.

D

A state's highway speed limits were 65 miles per hour in its flat land regions and 55 miles per hour its mountainous regions. To reduce traffic fatalities and combat the fact that most of the vehicles on state highways were exceeding posted speed limits, the state legislature proposed banning the use of radar detectors. Citizens in the mountainous regions of the state, where most of the state's highway fatalities occurred, generally supported the ban, but citizens in the flat regions of the state opposed the ban, so the legislature adopted a law banning use of radar detectors on any road with a speed limit below 60 miles per hour. A driver whose car was equipped with a radar detector lived in the mountainous region of the state but frequently drove to the state's flat region. While on a mountain highway with a posted speed limit of 55 miles per hour, the driver was pulled over by a state trooper for speeding. While approaching the driver's car, the state trooper noticed that the driver's radar detector was turned on. The trooper ticketed the driver for both speeding and illegal use of a radar detector. The driver challenges his ticket for use of the radar detector, arguing that it is unfair to allow people in the flat lands to use radar detectors while prohibiting residents of the mountainous region from using them. Which of the following statements is correct regarding the burden of proof in such a case? A) The state will have to prove that the ban serves a compelling state interest. B) The state will have to prove that the ban is rationally related to a legitimate state interest. C) The driver will have to prove that the ban does not serve a compelling interest. D) The driver will have to prove that the ban is not rationally related to a legitimate state interest.

D

Congress determined that bees were important to agricultural and consumer interests. Therefore, it passed a statute requiring beekeepers to be licensed, regulating the standards for maintaining hives, and establishing safety standards for the removal of honey from hives. Congress appropriated $50 million for use by the Department of Agriculture to enforce the act. A taxpayer has challenged the validity of the act, arguing that the federal government lacks the power to legislate with respect to bee-keeping and so is unconstitutionally spending tax revenues. What is the most likely disposition by the federal court? A) The court will determine the validity of the statute if the taxpayer can show a direct benefit from a determination that the act is unconstitutional. B) The court will determine the validity of the statute if the appropriations in support of the act can be considered spending under Congress's power to tax and spend. C) The court will dismiss for failure to state a cause of action. D) The court will dismiss for lack of standing.

D

Congress has enacted a law providing that all disagreements between the United States and a state over federal grant-in-aid funds shall be settled by the filing of a suit in the federal district court in the affected state. The law further provides: "The judgment of the federal court shall be transmitted to the head of the federal agency dispensing such funds, who, if satisfied that the judgment is fair and lawful, shall execute the judgment according to its terms." Is this law constitutional? A) Yes, because disagreements over federal grant-in-aid funds necessarily involve federal questions within the judicial power of the United States. B) Yes, because the spending of federal monies necessarily includes the authority to provide for the effective settlement of disputes involving them. C) No, because it vests authority in the federal court to determine a matter prohibited to it by the Eleventh Amendment. D) No, because it vests authority in a federal court to render an advisory opinion.

D

Due to budget shortages and a critical need of funding to fight a war, Congress enacted a $25 tax on each person flying into an airport in the five most popular vacation destinations in the country, as determined by Congress. The tax was implemented, and officials in the five destinations were outraged, fearing that the number of vacationers to the taxed destinations would decrease due to the tax. If the tax is challenged in federal court by an official with standing, is the most likely result that the tax will be held constitutional? A) No, because it makes it significantly more difficult for persons to travel between the states. B) No, because the tax unfairly discriminates against certain vacation destinations by taxing them and not taxing other, similar vacation destinations. C) Yes, because the tax is necessary to achieve a compelling government interest. D) Yes, because Congress has plenary power to impose taxes to raise revenue.

D

Small, prolific mussels called zebra mussels were first introduced into the Great Lakes by a foreign cargo ship. They became a serious problem because they attached themselves to smooth, hard surfaces, and often clogged water intake pipes. Congress determined that zebra mussels posed a great threat to the economic welfare of the Great Lakes region and passed a statute requiring all Great Lakes water intakes to be coated with a special chemical compound that repels zebra mussels. Studies by biologists at a major state university showed that while the special chemical compound that the federal government has required was effective, it also was toxic to other aquatic life. The biologists recommended that Great Lakes intake pipes be coated with a less toxic and less expensive copper-based paint. On the basis of those studies and the recommendation, three Great Lakes states adopted laws permitting municipal water districts to coat their intake pipes with copper paint. Can municipalities using copper-based paint on their intake pipes successfully be prosecuted for violating the federal law? A) No, because the Tenth Amendment prevents Congress from interfering with integral government functions. B) No, because the municipalities are taking effective steps to combat zebra mussels in compliance with the spirit and purpose of the federal law. C) Yes, because Congress is in a better position to regulate the entire Great Lakes region than the individual states. D) Yes, because Congress may adopt laws regulating navigable waters.

D

The President pardoned a drug dealer who was convicted on federal drug charges. After that unpopular pardon, Congress passed a law (over the President's veto) providing that the President can pardon individuals convicted for drug offenses and sentenced to prison only after they have served one-half of their minimum term. After the law was passed, the President issued a pardon to another person who had been convicted of a drug offense. The offender had served less than half of her minimum term. The warden of the prison in which the offender was held refused to release the offender, citing the legislation. The President then directed the attorney general to file a suit to force the warden to recognize the pardon and release the offender, arguing the legislation is unconstitutional. Is the court likely to find the legislation constitutional? A) Yes, because Congress has power to pass legislation to create federal crimes. B) Yes, because Congress has power to enact laws necessary and proper to its enumerated powers. C) No, because the law improperly restricts the President's inherent power as chief executive. D) No, because the power of the President to pardon for federal crimes is explicitly granted by the United States Constitution.

D

To prevent the Supreme Court from whittling away the protections that previous Supreme Court decisions had created for individuals accused of crimes, Congress passed a law eliminating from Supreme Court jurisdiction all cases in which a state supreme court has decided that a defendant's federal constitutional rights have been violated. If the statute is held unconstitutional, what is the most likely reason? A) The determination of the extent of constitutional rights is precisely the domain of the Supreme Court. B) Congress's power to limit jurisdiction applies only to cases originating in the federal courts. C) To be effective, the action taken by Congress here would require a constitutional amendment. D) Congress may not eliminate all avenues for Supreme Court review of issues vested within the judicial power of the federal courts.

D

To reduce incidents of violence among male gang members at a youth center, a city passed an ordinance forbidding any male between the ages of 13 and 19 to enter the center unless accompanied by a female. An 18-year-old male who was refused admission at the center because he was not escorted by a female filed suit in federal court to strike down the ordinance as unconstitutional. Should the court find the city ordinance constitutional? A) Yes, because in these situations the government is performing a parens patriae function. B) Yes, if the city can show a rational relationship between the ordinance and maintaining order at the center. C) No, if the plaintiff can show that the ordinance was not necessary to promote a compelling government interest. D) No, unless the city can show that the ordinance is substantially related to important government objectives.

D

Which of the following questions is the BEST answer with respect to abortion? A) As a result of the Supreme Court's opinion in Dobbs v. Jackson, abortion is now illegal throughout the country. B) A state law banning abortion after 4 weeks of pregnancy, presumably before many women would know they are pregnant, would be unconstitutional as a violation of Substantive Due Process. C) A law criminalizing abortion as a result of rape, incest, or when it endangers the life of the mother, would be unconstitutional. D) Currently, a law providing for the death penalty or life imprisonment for any woman having an abortion, or any doctor assisting her, would be constitutional.

D

Which of the following statements is the BEST answer regarding state and federal constitutions and judicial review? A) Issues based entirely on interpretation of a state constitution with no conflict with the federal Constitution can nevertheless be reviewed by the US Supreme Court. B) If the federal Constitution banned same-sex marriage, a state constitution could still provide a right to same-sex marriage on the basis of an independent and adequate state ground. C) If the federal Constitution banned same-sex marriage, a state constitution could still provide a right to same-sex marriage on the basis of an independent and adequate state ground. D) If a state constitution legalized polygamy (marriage to more than one spouse), a federal court could not rule it unconstitutional unless the Constitution itself prohibited polygamy.

D

Which of the following statements is the BEST answer regarding the Slaughterhouse Cases? A) The Slaughterhouse Cases held that all US citizens were entitled to the rights provided by the Constitution's Bill of Rights. B) The Slaughterhouse Cases effectively eliminated the 13th Amendment from the US Constitution. C) The Slaughterhouse Cases eliminated all slaughterhouses in the city of New Orleans. D) The Slaughterhouse Cases limited the rights protected under the Privileges or Immunities Clause to those rights that were already protected prior to the passage of the 14th Amendment.

D

Which of the following statements is the BEST answer with regards to Parents Involved in Community Schools v. Seattle School Dist. No. 1 (involving Seattle's and Louisville's efforts to voluntarily integrate their K-12 public schools? A) The Supreme Court held that the Seattle and Louisville plans were constitutional because they were voluntarily adopted. B) The Supreme Court held that the Seattle and Louisville plans were unconstitutional because they were voluntarily adopted by the school districts. C) The Supreme Court held that the Seattle and Louisville school districts were not de facto segregated. D) The Supreme Court held that the school district plans were unconstitutional because race was impermissibly considered when designing the desegregation plans.

D


Kaugnay na mga set ng pag-aaral

NCTI FTTx Basics Conventional Exam Review

View Set

Milady- Chapter 5: Infection Control

View Set

中級から上級への日本語 語彙クイズU1

View Set